Well Logging Notes + Workbook
Well Logging Notes + Workbook
When we speak of a log in the oil industry we mean “a recording against depth of any of the
characteristics of the rock formations traversed by a measuring apparatus in the well-bore.” The logs we
shall be discussing in this book, sometimes referred to as “wireline logs” or “well logs”, and are obtained
by means of measuring equipment (logging tools) lowered on cable (wireline) into the well.
Measurements are transmitted up the cable (which contains one or several conductors) to a surface
laboratory or computer unit. The recording of this information on film or paper constitutes the well-log.
Log data may also be recorded on magnetic tape. A large number of different logs may be run, each
recording a different property of the rocks penetrated by the well. Wireline logging is performed after
an interruption (or the termination) of drilling activity, and is thus distinguished from “drilling-logs’’ (of
such things as drilling rate, mud-loss, torque, etc.) and “mud-logs’’ (drilling mud salinity, pH, mud-
weight, etc.) obtained during drilling operations.
Geology is the study of the rocks making up the Earth’s crust. The field of geology that is of most
importance to the oil industry is sedimentology, for it is in certain sedimentary environments that
hydrocarbons are formed, It entails a precise and detailed study of the composition, texture and
structure of the rocks, the color of the constituents, and identification of any traces of animal and plant
organisms. This enables the geologist: (a) to identify the physical, chemical and biological conditions
prevalent at the time of deposition; and (b) to describe the transformations that the sedimentary series
has undergone since deposition. He must also consider the organization of the different strata into
series, and their possible deformation by faulting, folding, and so on. The geologist depends on rock
samples for this basic information. On the surface, these are cut from rock outcrops. Their point of
origin is, obviously, precisely known, and in principle a sample of any desired size can be taken, or
repeated. Sampling from the subsurface is rather more problematic. Rock samples are obtained as cores
or cuttings.
USES OF LOGS
A set of logs run on a well will usually mean different things to different people. Let us examine the
questions asked–and/or answers sought by a variety of people.
The Geologist:
1
'' How good a well is it?
'' What are the reserves?
'' Could the formation be commercial in an offset well?
The Geophysicist:
2
WELL LOGGING:
It is the record of the sub-surface formation in the form of the graph. The graph is between the
depth and some property of the sub-surface formation or any parameter of drilling (such as WOB).
The art of reading and understanding the graph or record of the log is called interpretation.
It helps to determine:
Lithology
Porosity
Saturation
Permeability
Extent of mud invasion into the formation
Bore hole condition
Electrical properties, gamma-ray properties
Density of formation
Image of well bore
Fluid types
Differentiate between shale and sandstone
Any log does not give the actual value of the property of the formation such as porosity, permeability,
saturation but with the help of log and using relevant formula we come to find that property of the
formation. The first logging that was ever done was by 2 persons from Schlumberger in 1924 and the log
that was run was Resistivity Log.
3
SP (SPONTANUOUS POTENTIAL LOG) LOG:
It is the most basic and simplest log. It works on the principle that a circuit is completed
between the well bore and the formation and current is passed due to which the electrical properties of
formation are determined. SP log is used to determine the Permeable zone.
REGORSITY:
It is the deviation of hole from its diameter that takes place due to fall of the formation into the
well bore and is measured with caliper log. The second figure shows the arrangement of logging tool
when running upward towards surface as for downward, these will be stuck into the irregularities of the
formation shown in figure 1.
Before logging we should know the environmental effects (invasion of mud) that affect the well
logging.
It tells whether the formation is shale or sandstone. In shale or clay it gives high value as shale
contains radioactive elements and gamma ray log detects the radioactivity of formation. Main
radioactive elements are potassium, thorium and uranium. Potassium and thorium are mostly present in
shale. In sandstone it gives less value most of the time due to lack of radioactive minerals but when the
radioactive minerals got high in percentage, it also gives high value in sandstone and difficult or
4
sometimes impossible to differentiate between the shale and sandstone through this log therefore we
never rely on the results of one log.
If some part of sandstone contains high contents of radioactive minerals, then section will be recognized
as shale and not sandstone like in this case:
Small percentages of clay dispersed in sand can cause significant increases in radiation
A. Maximum value of GR (Gamma Ray log) indicates pure shale and minimum value of GR indicates pure
sandstone. Now, to calculate volume of shale in sandstone we have following formula
5
MUD INVASION:
It is the loss of liquid part of the mud into the formation also called as filtrate loss and solid parts
of mud deposited on the wall of well bore forms the mud cake and it happens wherever the formation is
permeable. Mud invasion is determined using Resistivity log as we have already studied that flushed,
transition and virgin zones are formed due to invasion of mud which is differentiated using resistivity log
and then by using Archie formula we find the saturations of filtrate.
6
Saturation
Quick look technique
Mud logging includes drilling parameters such as WOB and ROP etc. Advanced logging includes logging
while drilling and drilling is not stopped.
LOG
A log is a visual analytical tool which serve for geologists, mud engineer, petroleum engineers and
service engineers responsible for drilling.
AIM OF LOGGING
We conduct logging for tabular and graphical representation and for sub-surface features encountered.
TYPES OF LOG
Driller log
Sample log
Mud log
Electrical log
Resistivity log
Gamma ray log
SP log
Neutron log
Induction log
X-PLOTS (CROSS-PLOTS)
Neutron porosity measures for limestone which is standard but for sandstone we have to do correction.
7
Porosity from density log
It measures apparent hydrogen content of formation. Neutron has same atomic number as that of
hydrogen. Log calibrated and scaled in Limestone porosity units and to calculate porosity of sandstone
and dolomite, we have to apply correction.
Sand + 2-5%
Dolomite – 2-5%
8
DENSITY-NEUTRON CROSSPLOTS
Neutron log gives value of porosity 20 which is for limestone but porosity for dolomite and sandstone is
Sandstone = 25%
Dolomite = 13%
DENSITY-NEUTRON COMBINATION
9
From the combination of two logs, we have some thumb rules:
If density log and neutron log overlap then out of three one will be present
Water filled limestone
Oil filled dolomite
Shaly sand
10
ENVIRONMENTAL EFFECTS
Resistivity logs have large environmental effects due to their large volume of investigation
Invasion
Shoulder Bed Effects
Thin Bed (Resolution)
ACOUSTIC LOG
It measures speed of sound in the formation in microseconds/ft. It is used to determine porosity and
lithology. Speed of sound is more in solid then in liquid and then in gas.
Gas-oil contact can be determined through acoustic log. It is because speed of sound in gas is
slow as compared to liquid so sound waves take large time to pass through gas.
Porosity can be determined from acoustic log using Wylies formula
= 189μs/ft (water)
= 55μs/ft (sandstone)
11
SOLUTION
By comparing three logs, we can predict different parameters. Condition (1) suggests wet reservoir as
shale content is less, less resistivity (water) and high porosity value. Condition (2) suggests tight
reservoir as shale content is less, high resistivity (hydrocarbon) and less porosity value. Condition (3)
suggests potential hydrocarbon reservoir as shale content is less, high resistivity (hydrocarbon) and high
porosity.
IMAGE LOG
Image log is used to get the 2-D image by which we can check:
Formation
Casing leak
Perforation
12
13
INVASION AND RESISTIVITY:
Before taking any log we need to find the invasion of the mud into the formation means at what
extent the different zones are produced into the formation due to mud invasion as the mud invasion
effects the well logging. The extent of mud invasion can be determined through the resistivity and we
can distinguish among the different zones on the basis of the resistivity as mud invasion alters the fluid
distribution around the well bore so after finding resistivity we make an invasion profile that gives us the
clear idea of what happened into the formation as the result of mud invasion. So, before logging we
have invasion profile that tells us how much mud filtrate and what type of mud filtrate is present in the
formation. Mud invasion depends upon differential pressure between well and formation, fluid loss
from mud, mud cake permeability, length of time it is exposed to formation. With porosity it has inverse
relation although with permeability it has direct relation.
INVASION PROFILES
well Rxo
Rmc Rt
Rm
Initially in mud we had some resistivity, as we go towards wall of well bore, resistivity increases in mud
cake as in mud cake the water is removed and only solids are there, as we enter into the flushed zone,
the resistivity increases as now the water will be present in the pores rather than free water as was in
case of mud, so rock grains provide high resistivity so resistivity increases, now as virgin zone comes, the
resistivity declines as formation water contains more salinity and resistivity here declines. The horizontal
line does not show that resistivity in each zone remains same but it changes in each zone too. For
example in mud cake as we go deeper, the resistivity increases as more concentration of solids will be
there. Similarly as flushed zone progresses, resistivity in flushed zone decreases due to more dissolution
of salts. Now in virgin zone it becomes constant due to same properties of fluid throughout this zone as
no contamination is here. The above profile is general with no transition zone.
14
Uncontaminated zone
Flushed zone
Mud cake
Well bore
Rxo
Rmc Rtr
Rm Rt
Up to flushed the explanation is same as that in general profile, after flushed zone the transition
zone comes in which the mud filtrate concentration or saturation decreases and formation water
increases and resistivity decreases as formation water is more saline and conductive and then virgin
zone comes in which only formation fluid is there and resistivity remains the same. This is the case when
there is water bearing formation but when we have oil bearing formation then profile will be like this:
Flushed zone
Virgin zone
Annular zone
15
Up to flushed zone the explanation is same, after flushed zone if hydrocarbons are present in
the formation then after flushed zone as filtrate enters into further formation, the water saturation in
the hydrocarbon zone increases from critical or connate to some extent as water will displace oil so
resistivity decreases initially in the zone just after flushed zone (the resistivity of filtrate from the
starting of flushed zone up to this area continue to decrease as more and more salts continue to dissolve
in the filtrate from sub-surface formation and more good filtration occurs so that the mud solid particles
continue to remove to enhance the conductivity of filtrate as it passes by, so filtrate resistivity never
remains same) then resistivity increases due to presence of hydrocarbons which become constant in
virgin zone.
Some points:
If mud is water based and water bearing formation is there then no annulus will form and
Rmf>Rw/Rt
Rxo>Rt
If oil formation is there:
Rmc<Rt
If mud is oil base and water in formation then no annulus.
Annulus indicate hydrocarbon in formation.
If water base mud but HC+H2O in formation there will be annulus.
Problem:
A given fluid loss, differential pressure, mud cake, and exposure time indicate that 20 lit of
mud filtrate invades into a porous, permeable oil bearing formation, if formation thickness is, h= 10
ft, residual oil saturation Sor= 25% and bore hole dia dn=8in. Calculate dia of invasion, di=?; if
porosity is 40%.
Solution:
H=10ft=120in dn
V=20lit=20dm3 h
1dm3=1000cm3 di
( )
16
( ) ( )
( )
If we solve the above problem taking porosity as 10% and invaded volume as same as above (which
is not practically possible, less porosity means less permeability and less invaded volume to keep
invaded dia same) then equation 1 becomes:
( )
The information regarding the different parameters of borehole environment are to be listed in the
sheet which looks similar to as shown below, it includes following:
1- Logging company name means which company is running the logging tool.
2- Well name and field name, country and state.
3- Well identification, typed or printed means location of well as per country’s rule.
4- Exact well location according to and system used in state (section, township and range)
5- Other log services run in the well.
6- Permanent depth datum and its elevation example ground level or mean sea level.
7- From what depth the log is measured.
8- And many other parameters.
In 7 we have to write the reference from where the log depth is measured such as KB
(Kelly bushing), DF (derrick floor), or ground level and it height above our permanent datum.
In 8 we have to write the reference from where the drilling depth, depth of formation
that has drilled is to be written.
Run number means how many times the log has run.
Logger and driller depth may not be the same due to different ways of measuring depth,
driller measure depth by drill string length while logger uses flexible wire.
Bottom and top log interval means up to which depth log has taken and from which
depth it was started respectively.
Casing driller and casing logger means the depth of last casing shoe and casing size given
by driller and logger. By comparing this and above point we can conclude that logging is done
in open hole (for open and cased hole we need different equipments).
17
Source of sample means from where the sample has taken either pit or return line. API
suggests that sample is to be taken from mud return line immediately when the circulation is
stopped and logging tool is ready to run after tripping out.
Rm, Rmf, Rmc are the resistivities and measured temperature means temperature of
sample when it was taken and its resistivity was measured. Similarly BHT means bottom hole
temperature. The resistivity at BHT is measured mainly from chart. Maximum temperature
means maximum temperature in sub-surface recorded during running.
Source of Rmf and Rmc can either be sample (M) or calculated from chart (C).
Time when last circulation was stopped for logging.
Logger at bottom means the time when the logger reaches the bottom hole.
Equipment and location means equipment number and truck’s base of operation.
18
19
20
Temperature Logging
We know that as we go deeper, the temperature of the subsurface formations tends to increase, due to
heat flow from the interior core. The average increase in temperature per unit depth in any formation is
called geothermal gradient.
This gives the average geothermal gradient from surface up to the depth that is put in but in practice,
this is not the case. Every formation depending on its lithology has its own ability to transmits heat from
it and thus possess a different geo thermal gradient.
For example, salt has the ability to transmit or allow more heat to flow through it so heat being not
trapped in this formation results in low geothermal gradient whereas shale acts as blanket and trap heat
in it so its geothermal gradient is high. It means the lithologies having high thermal conductivity have
less geothermal gradient and vice versa. The gradient also depends upon the heat flow.
The graph between temperature and depth is not always linear but it is linear when the formation is
homogenous having no other mineral impurities.
21
The temperature variation along the depth (geothermal gradient) and along with position in a basin can
be expressed in a map form using contours of equal geothermal gradients.
Every logging tool that is run should have to measure at least the maximum temperature such that the
temperature in the bottom hole. BHT is commonly measured by strapping to the tool (logging tool, such
as density) usually three pressure resistant, maximum thermometers and the average was taken. The
thermometers are mercury filled such that mercury absorbs heat from mud and expands, there is
restriction in thermometer such as what we have in thermometers at home, so the thermometer once
reaches to a value never drop down. When the tool comes to the surface, we can see the reading of the
thermometer attached to the tool, for accuracy we install three thermometers so the average
temperature can be read out. In this way we can get BHT without running any separate tool that can
cause time and leaving the open hole for too much time is undesirable.
The temperature that the thermometer reads is not the formation temperature but the mud
temperature. Mud tends to be cooler than formation and when we stop circulation, the mud absorbs
heat from formation and its temperature tends to increase but it will take a very long time for mud to be
in equilibrium with formation temperature. During logging, the mud is given time from the event when
we enter the logging tool into the bore hole and as it reaches the bottom we pulled it out (the tool is not
allowed to be at bottom for long time), so when it reaches the bottom, it will read the maximum
temperature which will not be the formation temperature as the mud had not come in equilibrium in
this short time (time from the mud circulation has stopped or the first tool was run to the time when the
tool has tripped out).
So we need to correct the value of the maximum temperature that thermometer reads, the correction
is known as Honer Plot method. In this method, we measure the maximum temperature at least thrice,
one with every tool that runs into the formation, so we run three tools successively one after the other
so that the time for each tool in which the mud can come into equilibrium are different then we plot the
measured maximum mud temperatures against the factor dt/(t+dt) on semi log graph, where dt is the
time from when the circulation was stopped to the time that particular tool was tripped out (time given
to the mud to achieve equilibrium) and t is the time for which the circulation was continued after drilling
the last formation to clean out the hole. From readings of the three tools, we can get three points on the
graph, the graph is extended when the value of dt/t+dt becomes 1 and read the value of temperature
22
on y-axis which will be true formation temperature, the reason is that the formation temperature as by
Honer is given by:
Now if we evaluate ( ) then answer would be one, and we know that log (1) is zero so the
factor after minus sign becomes zero and then Tm will be equal to Tf, this is the mathematical reason,
the logical reason is that at dt tends to infinity the mud will come in equilibrium with formation
temperature.
If we need to calculate or measure the temperature variation in each formation after certain interval
continuously then separate instruments or logging tools are used as maximum reading thermometer can
only give maximum temperature in the subsurface and not the continuous readings.
One of such instruments is the temperature Sonde which mainly consists of platinum wire exposed to
borehole fluid. The resistivity of the wire depends on the temperature of the mud according to well-
known relationship so measuring the resistivity of wire using wheat stone bridge can give indirectly the
temperature of the subsurface. Least count is 0.5 degree Fahrenheit.
More advanced temperature tools use a semiconductor type thermometer called a thermistor. Electrical
properties of thermistor depend on the temperature. The instrument has least count of 0.005 degree
Fahrenheit and is more sensitive such that its response to a temperature change is much quicker.
23
USES OF TEMPERATURE LOGS
We know that temperature logging is meant to determine the sub-surface formation temperature,
either we determine the maximum temperature only such that BHT or measure temperature
continuously along the depth. So we have separate uses; such that uses of measurement of BHT and
uses of measurement of continuous temperature.
Maximum temperature in the sub-surface gives the idea of upper and lower limits of
temperature, which is needed because the various logging tools are sensitive to temperature and can
work efficiently only when the temperature is in certain range, if not then corrections are to be done.
For example temperature logs are used to calibrate the resistivity logs to standard temperature and
pressure.
24
We can also know the burial history curve with time, depth and temperature to predict
maturity as shown by following graph:
In order to understand above graph, consider two source rocks start depositing at same time in the past
during cretaceous age in the areas having same geothermal gradient of 30 degree centigrade per km.
The rate of deposition (depositional depth per unit time) of rock 1 is greater than the other. Such that at
75 years before today, rock one has deposited to 2km depth while that of 2 is merely up to 0.5km depth.
Since the geothermal gradients of both are same, therefore up to this time the temperature of rock 1
has reached to 60 while that of other is only 20. So hydrocarbons may have started to form in rock 1 at
this time but not in 2 as oil window has not reached there. The rock 2 will take much more time to make
its organic matter to be matured.
3- Bottom hole temperature also effects the cement setting time so it is important to evaluate it
before cement job.
If we there is gradual increase in the temperature along the depth during drilling (such that
almost constant geothermal gradient, and the temperature in the subsurface suddenly changes to a
wide variation then this means that some formation fluid has either entered into the well or has lost
into the formation. If there sharp increase in temperature above normal trend then it means that
some inflow has encountered as we know that formation fluid has more temperature as compared
to mud so when it comes into the well, the temperature will increase dominantly. Similarly in case of
any fractured zone that is encountered, the temperature will be decreased as the mud will enter
into formation and cause it to cool by absorbing the heat for some time.
25
Difference between wireline and slickline
26
CALIPER LOG
When we drill the well, it is our utmost priority that we should drill a well that is in circle in
geometry and has diameter equal to the bit diameter throughout its extent. But unfortunately this is not
the case in practice. The actual diameter of hole can be greater or smaller than the bit diameter in most
situations depending on the formation being drilled and drilling parameters. So, after we have drilled up
to specific formation, we need to check the diameter of the hole and the bore-hole geometry. So the
diameter of the hole and the geometry of the hole can be determined through caliper log. Beside these
two purposes caliper log helps to determine:
Bore-hole volume
Mud cake thickness
Porous and permeable zone identification
Hole deviation
The caliper logging tool has arms (two, four, six, eight, etc) that are pushed against the bore-hole wall.
The crests and troughs on the wall make the arm to contract and expand, the arm is attached to variable
resistance so the movement of the arm changes the resistance and the electrical response to the surface
recording computers and hence the log is prepared. Caliper log can also be run with other tools such as
neutron density tool etc.
The two arm caliper log has only two arms at 180 degree to each other. These two arms give diameter
of the hole along only one axis. Since these arms can only give diameter along one axis only therefore
the two arm caliper cannot give the bore hole geometry as to find out the bore hole geometry we need
at least two diameters of the bore hole given by four arm caliper log. From the two arm caliper log, we
can determine where the hole is on-gauge, where there is cave, mud cake, sloughing hole, etc as shown
in the following diagram:
27
The two arm caliper log can either be represented in simple form or differential form. In simple form the
reference line is bit size or diameter but in differential form the bit diameter is taken to be zero and the
log gives the difference in diameter between the hole and the diameter.
If the log shows increase in diameter then it is the indication of washed out or caved zone. This
usually happens in unconsolidated formations that the formation around the well bore falls down.
If the log shows a decrease in diameter than the bit size then this is the indication of:
o Mud cake presence (if the log profile is smooth showing decrease in diameter). The mud cake
presence is the indication of good porous and permeable zone. Mud cake thickness is equal
to (bit size diameter minus caliper reading) divided by two.
o If the profile is not smooth it means that there is sloughing hole or tight spots present that
may occur due to water sensitive clay that swells in the presence of water and decrease the
hole diameter.
28
FOUR ARM CALIPER LOG
The four arm caliper log helps to determine the bore hole geometry as there are four arms or two pair
of arms. Each arm is at 90 degree with respect to the other. The two arms 180 degree apart form a pair
and give diameter reading along one axis (say 1-3) while the other pair gives the reading at the axis
perpendicular to the first pair (say 2-4), both readings are shown on the log and with these two
diameters we can show or imagine the bore hole geometry whether the hole is on-gauge and round,
oval and washed out, enlarged by a key seat or there is a break out as shown in figures. Similarly in 6
and 8 arm caliper, we will be having 3 and 4 bore hole diameter readings (as each pair will give one
reading) by which more appropriate geometry of the bore hole can be visualized. Four arm caliper tool is
also called dipmeter tool or bore hole geometry tool. Log can give us bore hole volume, hole deviation,
etc.
29
Three main types of elliptical bore hole have been recognized
such that key seats, washouts and breakouts.
30
The following graph shows that the hole was initially oval with major axis along caliper 1-3, the hole
after some depth become circle and then again oval with major axis along 2-4.
31
ELECTRICAL LOGS (RESISTIVITY AND CONDUCTIVITY LOGS)
This method is developed in 1919 and the first log was run in 1929. Electrical log is considered as plot of
certain electrical properties of the strata in contact with the bore hole.
Resistivity log is a measurement of formation resistivity i.e. resistance with the passage of
electric current
Conductivity tool measures formation conductivity i.e. an ability to conduct electric current.
Most rock materials are essentially insulators while their enclosed fluids are conductor. When water or
saline water that means conductor (low resistivity). When hydrocarbon in formation, resistivity is high
that means insulator. High resistivity value may indicate porous permeable hydrocarbon bearing
formation.
PRINCIPAL USE
to find hydrocarbon
use in petrophysical analysis and calibration
to study, lithology, facies and co-relation
LIMITATIONS
Resistivity tool cannot be run in oil based mud and fresh water based mud and can be run only
in conductive water based mud.
Induction tool cannot be run in conductive water based mud and run only in resistive mud.
Resistance Vs conductance:
Resistance is the ability of a material to impede the flow of current through it while
conductance is the ability to let the current pass through it, units are ohm and ‘S’ respectively. They
are dependent on material, temperature and dimensions of the material.
RESISTIVITY
Where; R=resistivity, r=resistance, A=area, L=length of resistor, it depends on the temperature, nature
of material and in case of solution, depends upon salinity as well. More temperature, less resistivity (due
to high K.E. of molecules), while more salinity more conductive due to large concentration of ions.
32
Unit:
MEASUREMENT OF RESISTIVTITY:
Resistivity logs have large environmental effects due to their large volume of investigation,
which are:
o Invasion
o Shoulder Bed Effect
o Thin bed resolution
33
34
35
TESTS TO MEASURE RESISTIVITY
First test is a direct measurement. A current is passed between two electrodes on a logging tool
and the potential drop between them provides resistivity.
Second test is indirect in that it measures conductivity. A current is induced in the formation
around the borehole and the capacity to carry the currant is observed. This carrying capacity is
the conductivity. Resistivity is simply the reciprocal of conductivity.
The simplified equivalent circuit for this device is shown in figure. Assume that there are numerous
spheres that are formed around electrode A with this electrode as their centre. The points locating
on the each sphere are assumed to have same resistivity R.
The current electrode A and potential electrode M are mounted on the sonde and lowered into the
borehole. Electrodes B and N are located at the surface far from electrodes A and M. The generator
induces a low-frequency, constant current I between electrodes A and B. The current will flow outwards
in spherical form creating equal potential at the surface of each sphere (as resistivity on the surface of
each sphere is assumed to be constant). There will be two potentials that will be measured, one is by
36
electrode M (it will be the voltage on sphere surface whose radius will be AM) and the other voltage will
be of sphere of radius AN detected by electrode N at surface, since the radius of AN is very large so its
potential will be very small to be neglected so the voltmeter measures the potential of electrode M,
knowing the potential on M, we can find the resistivity of the points on the sphere of radius AM that is R
and since the sphere will contain the points of formation so the resistivity will be of formation too.
( )
̅̅̅̅̅
̅̅̅̅̅ is sonde spacing and R is the resistivity of formation. The mid-point between electrodes A and M is
called inscription point. Normal device is said to have a radius or depth of investigation that is twice the
spacing ̅̅̅̅̅ Radius of investigation is the area of formation which is investigated by potential
difference.
The potential area and radius of investigation depends on sonde spacing. By changing sonde spacing,
radius of investigation changes, larger the spacing, more will be the radius of investigation but poor
resolution.
Since we have taken the assumption of uniform resistivity, but this is not the case so we generally
produce two logs one is the short normal with spacing of 16inch and radius of investigation of
approximately 2ft, it is used for correlation, for the evaluation of thin beds and for the location of bed
boundaries. The other log is the long normal of spacing 64 inch and radius of investigation of 10 ft, it is
used to over come the bore-hole effect and invaded zone effect and determine the true representative
value of formation resistivity Rt but the resolution of this log for thin beds is very poor (for more detail
page 209-210 of carl gatling).
LATERAL DEVICE:
This device is designed to overshoot the borehole and invaded zone, thereby measuring Rt. The
equivalent circuit is shown in the figure. Two pick up electrodes M and N are relatively close together
with A being some distance removed. The distance from A to the mid point of M and N, point O, is
considered the spacing, and will be referred to as OA. When current is passed through A (transmitter),
the equipotential spheres will be produced, the meter will give us the potential difference between the
spheres of radii AM and AN, such that ∆EMN, which indicates the voltage at point O such that the mid
point of M and N. The resistivity with this meter reading can be evaluated using the formula:
37
( )( )
( )
Where x1=distance AM and x2= distance MN which will be constant for the particular sonde having the
transmitter and these two receivers. If AO is sufficiently large then the borehole effects and the invaded
zone effects will be neglected and true formation resistivity Rt can be evaluated. The resistivity will
however be apparent unless necessary corrections are made. In practice, the spacing of the lateral
device is usually 19ft (18.8ft to be exact). It has more efficiency than that of normal device means it can
detect thin beds and it can overcome the effects of invaded zone and give good Rt.
38
voltage measured (AM2) will be called as 2 inch short normal with radius of investigation of 2 inch (here
radius of sphere is equal to radius of investigation). From the voltages the resistivities can be measured
by the known relationship, the resistivity measured by micro inverse is denoted by R 1x1 and that
measured by 2inch short normal is denoted by R2”.
The micro-inverse resistivity basically is the resistivity of mud cake Rmc (as it has small radius of
investigation as compared to 2 inch short normal) while the 2 inch short normal gives the resistivity of
flushed zone Rxo. Generally Rxo>Rmc.
If on the log, R1x1< R2” (it means that mud cake has developed that is giving the resistivity less
than the flushed zone so the formation is permeable).
If on the log, R1x1= R2” (it means that mud cake has not developed and both resistivities are of
formation, here no flushed zone will form as no mud invasion ha taken place).
If on the log, R1x1> R2” (This is the exception for example, if the measurement is obtained
opposite to salt water sand, in which some invasion has occurred and mud cake is also produced
then mud cake resistivity may greater than formation resistivity).
Micro log can be run alone as a tool or its tool can be attached with caliper arm, while attached to
caliper arm, it will remain attached to bore hole wall and bore hole effects will not introduce.
39
FOCUSED CURRENT DEVICES:
As the name indicates that in this device current is focused in particular zone of formation by
using guard electrodes. It includes number of devices depending on the number of guard electrodes and
are as follows:
It has three electrodes. Constant current is passed through the electrode A and same polarity
current and amount is passed through the guard electrodes G1 and G1’ due to which the potential
difference between G1 and G1’ or between any two points on the tool equidistant from A in the vertical
direction remains zero so as potential difference in the vertical direction remains zero so no current
from A can flow in upward or downward directions so the electric beams will be focused towards the
formation and will be concentrated in the horizontal direction and strength of the electric signals
increases in the particular zone and these beams can travel much horizontal distance into the formation
in that zone due to which the efficiency increases for measuring thin beds and more will be the vertical
resolution (means more efficiently it can detect thin beds but since the electric field is concentrated so it
can cover small area around the bore hole.
Equipotential spheres
A 1ft
G1’ 5ft
Equipotential spheres
40
PL: Proximity Log
CHARTS GIVING THE USES AND DEPTH OF INVESTIGATION OF VARIOUS RESISTIVITY TOOLS
INDUCTION LOG:
Since the basic principle of resistivity logs is to pass the current through the mud and the
formation and receive the potential drop in each zone to calculate the resistivity (Ohm’s law), but this
principle is only valid when we have conductive mud, but whenever we have oil based mud or we have
dry hole then we cannot run the resistivity log but then we have to use the principle of induction log
(Based on Henry’s law). In this log we give a potential difference to the transmitter device that includes
basically an inductor, the inductor in the transmitter device generates a magnetic field due to applied
voltage and AC current (primary Magnetic field), this magnetic field when passes through the mud and
the formation, produces an induced AC emf in the formation, which produces a secondary magnetic
field in ghe formation, this secondary magnetic field is detected by the receiver coil (an inductor), that
converts the magnetic field again into the potential difference from which the conductivity of the
formation can be calculated and then the resistivty from the relationship that R α 1/C. The depth of
investigation is 80cm.
41
Array Induction Tool (AIT):
This is a new tool consists of 1 emitter coil and four receiver coils. It is also detected by the raw
signal. Due to presence of multiple receiving coils, we can determine the conductivity of multiple zones.
Induction tools are important because they provide the only resistivity measurements in well drilled
with oil-based mud.
After discussing the resistivity measuring tools, we will be discussing the parameters that we find during
the resistivity log, these parameters are:
Rm = Mud resistivity
Rmc = Mud cake resistivity
Rmf = Mud filtrate resistivity
Rxo = Flushed zone resistivity
Rt = un invaded zone’s containing hydrocarbons resistivity (as we perform resistivity log in
reservoir area)
Rw = Resistivity of formation water
Ro = Resistivity of Rock 100% saturated with formation water.
Resistivity
measurement
Fluid Formation
resistivity Resistivity
42
FLUID RESISTIVITY MEASUREMENT
The value of Rm can be calculated either from log (as discussed in resistivity logging tool chart)
or it can be measured by taking the mud sample and find resistivity using the formula:
Knowing the container area (A) and length (L) and applied voltage (V) and measured current passing
through the sample (I), Rm can be evaluated, but since the resistivity is effected by temperature and the
resistivity calculated at surface may be different from that at subsurface due to difference in
temperature so the mud resistivity calculated at surface is to corrected to the bottom hole mud
resistivity using the formula:
( )
( )
R1 and T1 can be taken as resistivity and temperature at surface, while R2 and T2 are respectively at
bottom hole. The temperatures should be taken in Fahrenheit.
The mud filtrate resistivity can either be determined by taking sample and performing the same
experiment as done on mud or using the approximate relationship:
The mud cake resistivity can either be determined by logging (as discussed in logging chart) or
by using the relation:
1) Take the sample of formation water or prepare a brine of the same composition as that of
formation water (by dissolving the equivalent amount of NaCl in water, equivalent amount will
be discussed later), and find its resistivity by taking it into a container of area (A) and length (L),
voltage (V) is applied and current (I) is measured and resistivity is measured by using the
following relationship:
43
The resistivity measured should be caliberated to bottom hole temperature (application of BHT
measurement) using the relationship as discussed in mud resistivity.
2) The second method to determine Rw is to make a chemical analysis to formation water and
determine the solid contents and their amount in ppm (ni) in the formation water, once the
individual amounts are determined, we sum them up to get total solid contents in the sample,
locating the total solid on the x-axis of the following graph, we will determine the factors or
multipliers for each solid (ki), multiplying each solid by the factor makes it to be converted into
equivalent amount of NaCl and in the end summing un all the equivalent amounts we get total
solid concentration in the equivalent amount of NaCl such that we use the formula:
We convert the total solid contents (concentration) into equivalent amount of NaCl because the chart
we are given for resistivity measurement has NaCl concentration as reference, so in order to use the
44
chart (as shown below) we need to know the salinity of water in terms on NaCl, so we had to convert all
into equivalent amount of NaCl.
45
Example:
Given Calculated
Ion ni (ppm) ki (ni)(ki)
Ca 419 0.8 335
Mg 1304 1.04 1350
Na 10710 1 10710
K 390 0.91 355
HCO3 146 0.25 36.5
SO4 2690 0.4 1076
Cl 19350 1 19350
Br 70 0.44 31
∑ ∑
Now taking NaCl salinity as 33243 ppm and temperature of 60oF, from the chart we can see the
resistivity, Rw = 0.24.
46
3) Another method to determine the conductivity of formation water is to use the following
formula:
Where Cs is the conductivity of the solution (brine), nt is the total solid contents (sum of ni)and
Ci is the conductivity of each solid which can be located in the following graph. The graph can be
used in the same manner as was used to locate the multipliers in the previous example such
that knowing the value of nt (on x-axis), the values of Ci for each solid can be determined.
47
The conductivity of the solution is 17.610 and this is the conductivity at 18oC or 64.4oF as the
chart of conductivity is given at this temperature. Since the resistivity is reciprocal of
conductivity so Rw = 1/17.610 = 0.057 at 64.4oF. Now if we check the NaCl concentration at this
conductivity (17.6) from the same graph then it will come to be 166000 ppm that is the
equivalent NaCl concentration of the brine or sample. And if we want to cross check the answer
then at temperature of 65oF and NaCl salinity of 166000 ppm, the resistivity of brine from the
chart discussed in method 2 comes to be 0.057 such that same as above so both methods are
accurate. In this method the resistivity will come at 64.4oF so it should be converted to BHT.
The value of Ro is calculated from the formation resistivity factor (F) given by:
Since the value of Ro will always be greater Rw therefore the value of F will always be greater
than one. We have already discussed how to calculate Rw, in order to calculate Ro from above
equation we should calculate the value of ‘F’ for the given formation. Formation resistivity
factor depends on turtosity and porositiy of the formation and is given by:
The above equation shows an inverse relation between ‘F’ and porosity, .ore the value of
porosity, less will be Ro (Rw remain constant as salinity is not changed) so less will be ‘F’. Do the
derivation of above relation from page 2 of zaki.
48
The equation of formation resistivity factor given above is not practical because the turtosity
factor is impossible to determine so scientist have done much work to derive relation between
‘F’ and other rock properties. In this aspect Archie gave a relationship between ‘F’ and porosity
as:
In the Archie’s work Winsauer et al made a modification and gave a relation as:
( )
In above equation, m is the cementing factor and depends upon the rock consolidation and has
values in the range of 1.14 to 2.52 while the value of ‘a’ lies between 0.35 and 4.78.
For very hard compacted limestones ‘m’ has value of 2.4 etc.
In order to determine ‘F’ from equation 1, we should know the values of a, m and porosity. The
porosity can be determined from logs and as well as from core sample retrieved. The values of
‘a’ and ‘m’ can be determined by having many core samples, we will determine the values of F
for each sample by F= Ro/Rw; first we will determine Rw of the solution with which the core
samples are to be saturated (the composition of brine should the same as of formation water to
get good results), then saturate each sample with brine of Rw and find Ro (Rw will remain
constant for all samples) to find ‘F’ of each sample, also we will determine porosity of each
sample by core analysis and then as we know that:
( ) ( ) ( )
Now if we plot the data of various core samples of ‘F’ and porosity on log-log graph paper then
slope of the best fit line will give us the value of ‘m’ and and the y-intercep will give the value of
‘a’ as shown in the following figure:
49
( )
( )
50
When we do not have core samples then we can use the following relationships to determine
approximate value of ‘F’, the following relationships are all free from the parameters ‘a’ and ‘m’
and only porosity is involved. Each relationship is derived from the specific number of samples
tested in the laboratory:
RESISTIVITY OF ROCK SATURATED WITH HC AND PARTIALLY SATURATED WITH WATER (RT):
The value of Rt can be determined by logging of the un invaded zone of reservoir formation or
determining the resistivity of the freshly prepared core achieved from the reservoir using the formula:
Once Ro and Rt are determined, we can determine the connate water saturation in the HC formation
using the Archie equation for clean formations as:
( ) ( )
( )
( )
51
The ratio Rt/Ro is called resistivity index IR.
The value of n depends on the rock type or the manner in which the pores are connected. The value of
‘n’ can be determined in the laboratory with a sample or samples of the formation of interest. The
resistance of the samples are first determined at different values of Sw. The resistance data are then
used to calculate Rt and IR. A log-log graph between IR and Sw is linear, the slope of the line defines ‘n’.
In practice the determination of value ‘n’ is very delinate(reason on page 12 of zaki), for clean
consolidated sandstone formations, the value of ‘n’ appears closed to be 2 so we mostly take n=2.
( )
Since
Since in case of 25% porosity, the values of ‘F’ calculated are approximtely same so any value can be
taken, however in the case of 15% porosity, the value 36.6 shows wide variation from all other values
and moreover it is calculated from humble relation that is based on the least number of samples so we
neglect this value and can take any other value or their mean.
52
Difference between wireline and slickline
53
EXAMPLE:
Read the slides from 37-44 and 28 that Sir has given.
54
Invasion Corrections to the Dual Induction-Focused Log
Two charts are provided to correct the dual induction-focused logs for invasion effects. The first chart is
used where Rt < Rxo, and the second chart is used where Rt > Rxo. A microresistivity device, such as the
microlaterolog, is recommended for the second chart, as Rxo is part of the required input to the chart;
i.e., the focused measurement is not adequate for the specified conditions.
This chart provides a method of obtaining Rt from Dual induction-Focused log readings where Rt is less
than RXO.
55
The first chart is entered with a ratio RFOC/RILD on the ordinate and a ratio RILM/RILD on the abscissa. The
example on the chart shows entries of 10 and 1.4, which cross one another at a point where di = 39 in.,
Rxo/Rt = 18.5, and Rt/RILD = 0.95
This calculation demonstrates that RILD is not seriously affected by an apparent 39-in. (1.0-m) invasion
diameter.
SECOND CHART
56
This chart provides a method of obtaining Rt from the Dual Induction-Focused Log readings where Rt is
greater than RXO. RXO is determined by Micro Laterolog.
The second chart is entered with a ratio RILD/Rxo on the y-axis and RILD/RILM on the x-axis with example
values of 4 and 2, respectively. It can be assumed that Rxo is microlaterolog resistivity, and it should be
corrected for mudcake thickness if necessary. The example illustrates the following output results:
Therefore,
57
SPONTANEOUS OR SELF-POTENTIAL LOG
It measures the natural electrical potential between borehole and formation with an electrode in the
borehole and a reference electrode at the surface. No artificial currents are applied and only natural
potential is measured. The measured voltage is called potential spontane’s or spontaneous potential
(SP). The voltages are generated mostly by differences in salinity between the formation waters and the
mud filtrate.
Principal uses
Principles of measurement
In oil field wells, two fluids concerned are mud filtrate and formation water. Salinity is present in both
and one has high concentration of NaCl and other has low concentration. Ions move from high to low
58
concentration. If the ions move from formation water to mud filtrate then log gives negative value and if
ions move from mud filtrate to formation water then log gives positive value.
General mechanism
Two electrodes M1 (moving electrode) and M2 (earthed electrode) are in borehole and at
surface respectively.
It shows potential difference in permeable formation as compare to impermeable formation.
The deflection is measure at surface in millivolts (mV).
The pores in formation contain connate water having salinity behaves like a natural battery
(contain concentrated NaCl).
Ions movement develop current between formation NaCl and mud
SP MECHANISM
Electrochemical potential
Electrokinetic potential
ELECTROCHEMICAL POTENTIAL
Electrochemical potential is created when two solutions of different salinity concentrations are in
contact. It is composed of:
Diffusion potential arises when mixing of two solutions takes place by ionic diffusion. Cl- ion is both
smaller and more mobile than the larger, slower Na+ ion. The ions mix (diffuse), therefore, at unequal
rates, creating a charge separation. Cl- ion mixes the quickest, thus increasing its saturation in the more
dilute solution. A potential is created between negatively charged dilute solutions with excess Cl- and
the positively charged, concentrated solution with excess Na+.
59
A porous permeable sand membrane is present at the center and on right and left sides dilute and
concentrated solutions are present respectively. From concentrated side, Cl- ion moves towards dilute
side more because its ionic size is less and it is mobile as well. Hence, concentration of Cl- ion increases
in dilute solution. A potential is created between negatively charged dilute solutions with excess Cl- and
the positively charged, concentrated solution with excess Na+ known as diffusion potential.
Shale potential
Shale potential arises when concentrated and dilute fluids are in contact across a semi-permeable
membrane. Clay minerals which form shale consist of layers with large negative surface charge (SiO2-).
Due to charge similarity, negative chloride ions effectively can’t pass through negatively charged shale
layers, while positive sodium ions pass easily. Hence, concentration of Na+ ion increases in dilute
solution. A potential is created between positively charged dilute solutions with excess Na+ and the
negatively charged, concentrated solution with excess Cl- known as shale potential.
60
Shale potential is larger of two electrochemical effects.
Electrochemical potential which is measured in borehole is the combination of diffusion potential and
shale potential. Consider porous and permeable sandstone penetrated by a borehole, mud filtrate in
this example is less saline then formation water.
Diffusion Potential
Sandstone bed is a porous and permeable membrane and mud filtrate is dilute solution in the example
so Cl- ion increases in dilute solution.
61
Shale Potential
Above sand, shale act as semi-permeable membrane and mud filtrate is dilute solution so, Na+ ion
increases in dilute solution.
The excess charge is therefore negative opposite the sand and positive opposite the shale.
More the difference between salinity of mud and formation, more potential difference will create and
more deflection will show on graph, mean more permeable.
If concentration of ions in formation is less than borehole then the log shows positive side. If mud
filtrate act as concentrated solution and formation fluid as dilute solution then according to diffusion
potential phenomenon, Cl- passes more from concentrated solution to dilute solution because of its
smaller size and mobility. Hence, in concentrated solution i.e. mud Na+ remains which creates positive
charge and hence, positive potential is observed.
ELECTROKINETIC POTENTIAL
If a solution is forced, by differential pressure, to flow through a membrane, an electric potential will
appear across the membrane.
A similar situation occurs when the mud filtrate flows through the mud-cake because of the differential
pressure between the mud column and the formation. This electrokinetic potential (Ek) is generally
small.
Limitation of SP log
Track 1 – log made on paper having equal distance lines (Examples are caliper and SP)
Track 2 – log made on semi-log graph
62
Log characteristics
SP bed resolution is bad. It does not tell where precise cut of shale and sandstone is. More
shoulder bed effect is present.
The thickness of formation should be 20ft to get maximum (full) deflection which is known as
Static Self Potential (SSP).
We can know whether we get SSP or not due to thin bed effect. When thin beds are present then we get
sharp end.
SSP is ideal case but some conditions causing aberrant SP values when SSP is not attained.
63
SP RESPONSE
QUANTITATIVE USES
Static SP or SSP (in ideal case), it is maximum possible SP with no shale. SSP is for quantitative evaluation
of Rw. Frequently SP does not show full deflection.
64
IDENTIFICATION OF PREMEABLE BEDS
SP deflection is caused by ion migration that requires the existence of some minimal permeability.
Deflection of SP is higher at the high porosity/permeability formation (away from shale SP).
Magnitude of SP deflection and hydraulic permeability has no direct relationship.
SP does not tell how much permeability is present.
65
SHALE VOLUME ESTIMATION
SP log is used to estimate shale volume when following conditions are fulfilled:
( )
Where;
( )
( )
( )
66
SP-WATER RESISTIVITY (Rw)
EXAMPLE 11.4
Thickness =32 ft
SP = 20 x 5 =100 mV
67
SOLUTION
( )
68
At SP = 100mV and Tf = 1720F
K=82
( )
For SSP
( )
( )
( )
For Rmf
For Rwe
( ) ( )
( )
For Rw
69
At ( )
For salinity
Salinity =70,000ppm @
70
GAMMA RAY LOG
WHAT IS GAMMA RAY LOG:
It is the record of the radioactivity of subsurface formation rocks against the depth.
WHAT IS RADIOACTIVITY:
Radioactivity is the process in which the unstable nuclei convert into stable nuclei by emitting
the high energy radiations such that alpha, beta and gamma rays. Radioactive elements are those
elements whose nuclei are unstable mainly because the binding energy in the nucleus such that the
energy that keeps the nuclei particles intact in the nucleus is small to keep the nucleus intact and this is
because the number of neutrons in the nucleus of these elements is far greater than number of protons
so due to less binding energy the unstable nucleus breaks into other smaller and stable nuclei and emit
different type of radiations which are as under:
Alpha Particle:
Beta Particles:
A beta particle originate when a neutron (n), in the nucleus breaks into a proton (P+),
and electron (e-) and a neutrino (ν). Neutrino is a neutral particle and its penetration power is
impossible to detect (very high). The proton formed remain in the nucleus so an increase in the atomic
number occurs (number of protons) but mass number remains the same (neutrons +protons).
Gamma Rays:
They are electromagnetic radiations similar to nature of visible light, radio waves, X-
rays.
Gamma rays are not continuous flow of energy but is a discreet series or bundles of
energy called quanta each series having a specific energy given by E=h ν.
They are basically photons that travel at very high speed such that the speed of light. It
is characterized by its frequency, wavelength and energy.
71
It has high penetration power and they are detected by the instrument.
Energy level is 0.01-0.1MeV.
The emission of gamma ray is independent of the emission of alpha and beta particles (which
are also independent of each other) due to which a nucleus is disintegrated into another nucleus and a
new element is formed. Due to emission of alpha and beta particles, the parent element converts into
the daughter element, now if the daughter element’s nucleus just produced is in the ground state then
no gamma rays will be generated but if it is in the excited state means the nucleons in the nucleus are at
high energy level rather than ground level then they emit one or more photons of distinct energy and
come into the ground state means due to emission of gamma rays, the element’s nature is not changed
but the energy of the system is lowered.
In the above example, the cobalt converts into Ni with emission of beta particle but the nickel
produced has its nucleus in the excited state so it emits two photons of distinct energy to come to
ground state.
There are energy levels in the nucleus occupied by nucleons just like energy levels of the atom
occupied by electrons. In the ground state of the nucleus, the nucleons are at the lowest possible energy
level but at excited state, they are at high energy level but they cannot remain here for infinite time so
they come to the ground state by emitting high energy photon. The energy is measured in MeV. One eV
is the energy required to move one electronic charge across a potential difference of 1 volt and it is
equal to 1.6E-19 Joules.
72
GAMMA RAY INTERACTION:
Now as we have discussed that the most appropriate radiation that is detectable to our instruments is
the gamma ray as it has good penetration power, but the fact is that the gamma ray that is produced in
the formation from the radioactive elements cannot retain its original energy up to the point when it is
detected by the instrument means a reduction in energy occurs as these rays travel from the inside of
the formation to tool. This reduction of energy in the gamma ray photons is due to the following
processes that occur in the formation and in the well bore, the possibility of occurrence of each process
depends upon the atomic number of the element emitting gamma ray and the energy of the gamma ray.
73
Pair production:
In this process, the emitted high energy photon strikes the nucleus of an atom and produces a
positron (e+) and negatron (e-). Since the energy equivalent to the mass of an electron is 0.511MeV so
the threshold (minimum) energy for pair production is 1.022MeV (but from chart it is 4.5 MeV). If the
incident photon has energy above this then as it strikes the nucleus it not only emits the positron and
negatron but the excess energy of incident photon will be divide among the two and will serve as the
kinetic energy for them, these two particles such that positron and negatron combines to give a
positronium, which then annihiliates into two 0.511MeV photons.
Compton Effect:
This process can occur if the incident photon is either produced from the pair production having
the energy in the Compton Effect interval or the incident photon is produced directly from radioactivity
and it has energy in this interval. In this process what happens that the incident photon strikes the
outermost electron of an atom and transfer some of its energy to it and ejects the electron out of the
atom, the energy of scattered photon will be less than that was originally there.
Photoelectric effect:
If the incident photon has very low energy either produced directly from pair production or from
the Compton scattering or it is the original gamma ray photon with energy in the interval of
photoelectric effect then this process will occur. During this process, the incident photon strikes the
inner orbital electron and ejects it with some kinetic energy. As a result of this no further photons are
produced so gamma ray has reached to its domain.
All these processes will occur whenever there are gamma ray photons produced in the formation due to
their interaction with formation other atoms, atoms in the drilling fluid and as well as in the tool if any
fluid is in there and as a result of these processes electrons are emitted from the atoms in the medium
with which gamma rays interact. In order to summarize all above discussion, we can say that the gamma
ray photons that were produced originally do not remain in their original form or possess lesser energy
when they are detected.
74
RADIOACTIVE ELEMENTS IN SEDIMENTARY ROCK:
Rocks contain varies amount of unstable elements which exhibit certain level of natural
radioactivity. In sedimentary rocks, radioactivity is due to three major elements like Uranium series,
Thorium series and potassium 40 isotopes. We have used the word Uranium series and thorium series,
because there is series of radioactive elements that are generated due to decay of uranium and thorium
that are further decayed to the most stable elements and there is series of gamma rays produced. The
series radioactivities of these two are as follows:
75
76
There is no series for potassium radioactivity as it is directly converted into stable atom in the first
reaction. That is why in the spectrum of gamma ray radiation for potassium we have only one line
whose probability is very high as we know potassium whenever undergo a radioactive process gamma
ray will be emitted. But in the spectrum of others there are several discreet lines showing energy of the
emitted radiation and probability of gamma ray emission (whether occur or not) in each step reaction of
the series.
77
In the above spectra, we have specified some peak values for each source that is the identification of
that source. We have taken these peaks because they are distinct from others. The above spectra are
showing discreet energies but when we take response or make spectrum of the radiations coming from
the formation the spectrum will be continuous because of the various processes of interaction that
occurs in the formation due to which many
photons loses their energy and we get
photons of various energies so a
continuous spectrum is produced. Small
peaks in between show the actual labeled
peaks are formed due to photons that have
lost their energies or due to raw photons.
78
79
Potassium is the primary source of radioactivity in sedimentary rocks. In all sedimentary rocks,
potassium is present in greater quantity as compared to others so it is measured in percentage, but its
contribution to the overall radioactivity is small related to its weight while Uranium and Thorium are
measured in ppm such that in traces but their energy is high so the overall energy emitted by each
source is approximately the same.
RADIOACTIVITY OF SHALE:
Radioactive elements tends to concentrate in shales, so generally it is assumed that high gamma ray log
reading means shale is present, which is not the case always. The radioactivity of shale is mainly due to
potassium that is largely present in the clay minerals like; ”Illite”, “Glauconite”, other clay minerals such
as Kaolinite and smectite does not have potassium. So a shale sample containing Illite in abundance
show high radioactivity but the shale containing Smectitie in abundance contains low radioactivity.
Potassium is chemically active so it is present in shales and other sedimentary rocks as chemically
bonded in the mineral molecules. Uranium and thorium are not chemically present in mineral
composition of rocks but they are present as elemental atoms among rock grains. The average presence
of potassium in shales is 2%-3.5% (average 2%), and this percentage helps in the indication of shale
through spectral log.
Thorium is much uniformly spread in shales and its average presence in shale is 8-18ppm
(average 12ppm) and this also helps in the indication of shale presence in the spectral log.
Uranium is not uniformly distributed in shales and also its presence is very low (2-6ppm) so it is
not a good indicator of shale, Uranium is widely concentrated in organic shales which show very high
value of radioactivity due to uranium.
80
GAMMA RAY LOG RESPONSES IN OTHER ROCKS:
Simple Gamma ray log gives the total radioactivity of the formation such that the radioactivity of
potassium, thorium and Uranium in combined form. The radioactivity is measured in API. It is made
standard such that there is well in University of Houston. The well contains specially mixed, high
radioactivity concrete bounded by equally special low radioactivity concrete. API unit is 1/200 of the
difference between the two radioactivities. The simple gamma ray log is usually recorded in the track 1
along with calipers. Scales are chosen locally, but 0-150 API is common. The tool is small and can be
combined with practically any other tool be it a resistivity or porosity device.
81
A high value of simple gamma ray log does not always indicate the shale presence but it is to be
confirmed trough other logs. This is because the radioactive elements are not only present in shales in
abundance, but they may be present in other rocks. For example potassium is not only present in clay
minerals but other rock forming minerals such as Feldspar, mica, etc. A sandstone formation may also
show high radioactive response due to presence of feldspar and is called as Arkose, moreover sand may
also contain high percentage Glauconite; similarly organic rich shales have high value of radioactivity
due to uranium present.
82
SPECTRAL GAMMA RAY LOG:
So for this we will be needing a log that will show the radioactivity of the three sources K, U, Th
separately in terms of their amount present in the formation, so by this we can easily recognize the
lithology such that which one is shale and which one sandstone because we know that in ordinary shale,
potassium is found in the range of 2-3%, Thorium as 12ppm, and we do not take the amount of Uranium
as shale indicator. We are emphasizing too much on shale because the major use of gamma ray log is to
calculate shale volume.
The spectral gamma ray log shows the amount of potassium in percentage, Uranium in ppm and
Thorium in ppm also.
83
The following spectral gamma ray log shows two curves; SGR and CGR. The SGR, or standard gamma ray
is the total contribution of the three elements in API units, that is, it is the same as simple gamma ray
log. The CGR stands for computed gamma ray curve, represents the contributions of only the thorium
and potassium in API units. Means the difference between CGR and SGR indicates the contribution of
Uranium. The SGR and CGR are plotted from the true spectral gamma ray on the right side showing the
amounts of each source separately and using the conversion factors:
1ppm U=8.09API
1ppm Th=3.93API
1%K=16.32API
Thus 3ppm of U contributes 24.27API units. The CGR showing the contribution ‘Th’ and ‘K’ shows the
improved clay volume and K and Th are good indicator of shale presence.
Ionization chamber
Geiger Muller counter
Scintillation counter
Proportional counter
The first two are based on the ability of gamma ray to ionize a medium such as gas through which it
passes. Scintillation counter is based on the ability of gamma ray to produce tiny flash light as it decays
into a crystal.
IONIZATION CHAMBER:
It consists of a steel chamber that is conductive and acts as electrode. An inert gas such as
Helium is filled in the chamber with high pressure from 1000-1500 psig. There is a conductive wire at
middle of chamber which acts as another electrode. A low potential of 100 volt is applied. When gamma
84
ray enters into the cell it ionizes the atoms of the gas, the ions under the influence of electric potential
flows and generate a small current that is to be amplified and is measured which gives the intensity of
radioactive ray. Due to its very low efficiency (5-10%) it is now obsolete.
The Geiger Muller counter’s principle is same as that of ionization chamber. It also consists of
steel chamber that is conductive and acts as electrode. In the chamber a low pressure gas is present and
we apply a very high voltage. Now as Gamma ray is entered, it ionizes some of the particles of gas, due
to very high applied potential, the ions that are just formed move with very high velocity towards the
electrode and during their way they also ionize the other atoms so an avalanche of ions move towards
the electrode and produce a uniform voltage pulse (as gamma rays are discreet energy waves so when a
particular energy wave enters, it ionizes and creates a potential, similarly another wave will come and
creates another potential so a pulsating voltage will generate. These pulses are counted and the
deflection is proportional to the number of pulses per unit time. This device measures the number of
pulses rather than current. Its main disadvantage is that its reaction is very slow and has poor measuring
efficiency.
SCINTILLATION COUNTER:
The sensor is a sodium iodide crystal that emits a burst of light when stuck by a gamma ray.
Within the PM (photo multiplier) tube, there is a photo sensitive surface that gives off electrons
when exposed to light.
The crystal is optically coupled to the PM tube and as the crystal absorbs a gamma ray, electrons
exit the photosensitive surface.
The PM anodes, each at a progressively more positive potential, attract electrons and give off
about 3 electrons for each electron striking the anode.
The pulse generated is amplified, shaped and sent to the instrumentation that scales and
statistically averages the data.
The time that is given to the number of gamma rays to enter into the PM tube is the time
constant.
85
86
87
88
The simple gamma ray sonde can be combined in many tools; it is run both centered in the bore hole
(sonic and resistivity tools) or against the bore-hole wall that is eccentered (density and neutron tools).
The eccentered tool gives large API value as compared to centered one because in case of centered one,
more Compton scattering occurs due to presence of mud.
LOGGING SPEED:
If the logging speed is high then the time constant will automatically be low and if the time given
for gamma rays to enter into the PM tube is less than the results are not favorable. The greater the time
constant is, more will be the number of counts in that time (so more good and appropriate will be the
result for “number of counts per second”) and hence the shoulder bed effect will be minimized and thin
bed resolution will enhance.
89
PRINCIPAL USES OF GAMMA RAY LOG
Quantitative use
In petrophysics, gamma ray log is used for determining shale volume (Vsh).
Qualitative use
As a depth correlation device, GR used to adjust the depth of the tool string in relation to the depth of
the formation being logged. Any depth offset could be observed by comparing the GR data logged from
different runs. Depth-shift will applied to match the recorded data depth, related to the reference data.
Its high vertical resolution makes it extremely useful for depth matching.
90
DEPOSITIONAL FACIES INTERPRETATION
91
LITHOLOGIC USE OF A GAMMA RAY LOG
STRATIGRAPHIC SEQUENCE
92
FOR CASING LEAKAGES, FRACTURED ZONE IDENTIFICATION & LOST CIRCULATION ZONE
Radioactive traces are mixed in mud and when this mud is circulated, traces go into formation or behind
casing if any casing leak, fractured zone or lost circulation is present and with the help of gamma ray log
we indicate it.
Quantitative evaluation of shale content using gamma ray log assumes that radioactive minerals other
than silts and clay are absent.
Where;
Set shale base line at maximum GR log reading GRsh and set sand base line at minimum GR log reading
GRcn. In zone of interest, take log reading equals to GR.
IGR obtained is corrected for the response in rocks of certain ages. If two formations of different
geological age have the same amount of radioactive shale per unit volume, the gamma ray log response
would be lower in case of more consolidated and dense formations. This is because gamma ray
absorption increases with the density of the medium through which it passes.
Curve 1 on the chart represents linear IGR response from 0 to 100% shale
Curve 2 is used to obtain shale volume for pre-tertiary age rocks (Mesozoic and Pelozoic)
Curve 3 is used to obtain shale volume for Tertiary and younger Quaternary formations
From chart
From IGR value obtained on y-axis, find volume of shale in % from x-axis after intersecting chat of
particular age.
From formulae
Volume of shale for different ages can be obtained from formula as well
93
( )
( )
PROBLEM 1
Sands and shales represent the interval shown on the problem 2 example. Determine Vsh for both
Tertiary and Mesozoic ages in zones A through I indicated by log?
SOLUTION
PROBLEM 2
A sequence of carbonates and shales is given. Determine Vsh for both Tertiary and pre-Tertiary
(Mesozoic) in zones A through F?
SOLUTION
94
CORRELATION OF DIFFERENT LOGS
95
SONIC LOG OR ACOUSTIC LOG
Sonic log is a record of the time required for acoustic waves to travel a given distance through the
formation that surrounds a borehole. The interval transit time designated as , the reciprocal of sonic
velocity. More the time , lesser will be the velocity.
BASIC CONCEPTS
Acoustic
These are velocities from maximum to minimum or travel time from minimum to maximum
Sand
Shale
Fluid
The more the fluid, the slower the speed and harder the rock, the faster the speed
SNELL’S LAW
sin(1 ) V1
sin( 2 ) V2
sin(1 ) sin( 2 )
V1 V2
1.
At Critical angle ,
96
V1 V
sin( c ) c arcsin 1
V2 V2
2.
Summary
1 2 1
V1
1 2 1
V2 1
2 1
Particle’s motion is in direction of wave propagation. P-waves can propagate in any type of material.
97
Shear or Transverse or S-waves
Particle’s motion is perpendicular to wave propagation. Presence of shear waves require medium to
possess shear strength hence, S-waves can propagate only in solids.
Acoustic (elastic) waves undergo interference, diffraction, reflection and refraction. Reflection and
refraction occur when a wave encounters a boundary separating two media with different elastic
properties. Part of the energy of incident wave is reflected and part is refracted. The incident wave may
be converted into other types of vibrations upon reflection or refraction. This phenomenon is called
mode conversion. For example, upon refraction, an incident compressional wave is partially converted
into a wave or waves of another type, such as shear wave.
Figure shows the geometry of the rays along which acoustic waves propagate. Figure shows a wave with
velocity , incident at angle , on a plane boundary separating two media of different elastic
characteristics. A wave with velocity is refracted into the second medium at angle . A third wave
with velocity is reflected back into the first medium at angle .
98
When then it is critical angle of refraction . The refracted wave does not
penetrate the second medium but travels along the interface at velocity . This critical refracted wave,
called the head wave, propagates energy back into the first medium as it travels along the boundary.
If the incident angle is greater than critical angle, no refraction will occur and the wave is totally
reflected.
A compressional wave travelling in medium 1 at a velocity will generate a compressional head wave
in medium 2 if its angle of incidence is critical .
A compressional wave travelling in medium 1 will generate a shear head wave if its angle of incidence is
critical .
The source of acoustic signal is usually centered in the borehole. Because fluids cannot sustain a shear
stress, the source (transmitter T) is a pressure transducer that generates compressional waves in the
99
borehole fluid. The received signal is a composite of several acoustic waves called a waveform. The main
components of the waveform are, in order of their arrival at the receiver
The first wave to arrive at the receiver is the compressional head wave, which is critically refracted at
the borehole wall. The wave begins as a compressional wave generated in the borehole by the
transmitter transducer. It encounters the borehole wall at an angle and is critically refracted into the
formation as a compressional head wave. The head wave travels along the borehole with the formation
compressional velocity, . As the head wave travels along the borehole fluid/formation interface,
energy is continuously radiated back into the borehole as compressional vibrations in the borehole fluid.
These vibrations, also propagating at a critical angle, , are sensed by the receiver centered in the
borehole.
The compressional wave arrives at the receiver first because of a combination of velocity and path
effects. Three velocities play roles in wave propagation in fluid-filled borehole: formation
compressional-wave velocity , formation shear-wave velocity , and fluid compressional wave
velocity . The compressional wave propagates the fastest in formation and this wave also has the
shortest path through the drilling fluid, where acoustic velocity is the slowest.
100
SHEAR HEAD WAVE
The shear head wave also begins as a compressional wave in the borehole fluid. It encounters the wall at
the critical refraction angle, , and thus refracts into the formation as a shear wave. Like the
compressional head wave, shear head wave travels along the borehole wall at a velocity at critical
angle . Because and because it has a longer path in the borehole fluid, the shear head wave
arrives at the receiver behind the compressional head wave. If , such as in extremely
unconsolidated formations, shear waves cannot be critically refracted along the borehole wall and the
shear head wave is not generated.
It is observed between the compressional and shear-wave arrivals. The leaky mode arises from
compressional waves incident at the borehole wall with angles between and . These
compressional waves undergo both reflection and conversion into refracted shear waves. The reflected
wave propagates in the borehole fluid as a conical wave. The amplitude of this mode decreases with
distance from the transmitter as the energy is leaked to the formation at each reflection. The spacing
between the transmitter and receiver could be designed long enough so that the remaining energy of
the mode at the receiver is too low to be detectable.
PSEUDO-RAYLEIGH WAVES
Pseudo-Rayleigh waves arise from the portion of wave incident on the borehole wall at angles greater
than critical angle of refraction of shear wave, . This wave undergoes total reflection and propagates
within the borehole as a conical wave.
101
STONELEY WAVES
Stoneley wave is a surface wave at the fluid/formation interface. Stoneley waves generated by the radial
flexing of borehole as the acoustic energy passes from the borehole fluid to the borehole wall in the
borehole are also called tube waves and guided waves. Velocity is always less than borehole velocity.
102
FULL WAVE ACOUSTIC SIGNAL AT THE RECEIVER
SONIC LOG
Sonic log provides a formation’s interval transit time, designated (the reciprocal of velocity).
( )
It is a measure of formation’s capacity to transmit sound waves. Geologically this capacity varies with
lithology and rock texture, notably porosity.
103
The conventional, general purpose sonic tools only measure compressional or P-waves, the first arrival.
A modern generation of tools is now available to measure the full wave train which includes the
compressional wave, shear wave and stoneley wave.
Given in
Value is called interval transit time
Velocity ( ) is the reciprocal of interval transit time
Log shows value in
PRINCIPAL USES
Quantitative
Qualitative
For the geologists, sonic log is used to indicate lithology, to help identify source rocks, normal
compaction and overpressure and to some extent fractures.
PRINCIPLES OF MEASUREMENT
The conventional, general purpose sonic tools measure the time it takes for a sound pulse to travel
between a transmitter and a receiver, mounted a set distance away along the logging tool.
104
The pulse measured is that of the compressional or P-wave and tool design enables the velocity of this
wave in the formation to be measured. Compressional wave is simply the fastest or ‘first arrival’.
Compressional wave is followed by shear and stoneley waves. These two waves are ignored in
conventional tools but in modern array acoustic tools, it can be fully measured.
A sonic tool transmitter typically produces source frequencies of between 10-40kHz or 10,000-
40,000 cycles/second.
At 10-20 kHz, acoustic wave has a wavelength of between 7.5cm (0.25ft) – 75cm (2.5ft) over the
velocity range of 1500m/s (5000ft/sec) – 7500m/s (25,000ft/sec)
Conventional sonic tools measure the reciprocal of the velocity of the compressional wave. This
parameter is called interval transit time, , or slowness, and is expressed in .
Where and are the slowness of matrix and pore fluid respectively. Average values of matrix
and fluid slowness are given below:
TABLE 1
105
Equation is derived with a simplified physical model and in unconsolidated formations, it overestimates
porosity. In such formations, equation is
( )
Where is a compaction correction factor related empirically to average slowness of adjacent shale,
, by
The presence of secondary porosity – i.e. vuggs and fractures – complicates the quantitative evaluation
of sonic logs. In such cases, above models yield an apparent porosity that reflects only the primary
(intergranular) porosity. Comparing this apparent value to that of total porosity, obtained from the
density or the neutron log, gives an order of magnitude of the secondary porosity.
SINGLE-RECEIVER SYSTEM
Early sonic tools were equipped with a single receiver. In this system, a pulse is initiated at the
transmitter situated at a distance from the receiver Ls (also called spacing).
The time measured, , is between the initiation of the pulse and the first arrival of acoustic energy at
receiver.
106
Figure shows the waveform at the receiver showing time measured by the single-receiver system.
( )
√ ( )
In deriving equation, we assume that tool axis is parallel to borehole axis but may be displaced from it
by a distance . If centralizes are used and tool is in borehole axis then . It is also assumed that
.
( )√ ( )
107
EXAMPLE 10.1 (ZAKI)
A compacted sandstone formation of interest, detected by an old single-receiver sonic log, displays a
slowness of 90 . Estimate the formation porosity if the bit is 7 3/8 inch, tool diameter is 3
inch, centralizers were used, and the tool spacing is 6 ft.
DATA
( )
SOLUTION
( )√ ( )
( )
( )√ ( )
108
EXAMPLE 10.2 (ZAKI)
Estimate the average porosity of the thick sand interval shown on the sonic log of figure
SOLUTION
Dual receiver system was introduced to remove the mud-path contribution from the response of sonic
tools. The tools consist of a transmitter and three receivers located 3, 4, and 6ft from the transmitter.
The transmitter emits acoustic waves at 10 waves / second. A two-receiver system can be viewed as a
very accurate stopwatch. The stopwatch starts when acoustic energy arrives at the first receiver and
stops when it arrives at the second receiver. The time indicated by the watch is the time required for the
sound wave to traverse a length of formation equal to the spacing between the two receivers. The
arrival times and at the two receivers can be expressed as;
109
Where, a through e are the paths of acoustic waves. Assuming that the hole size is constant and that the
tool is parallel to the borehole wall yields
Figure shows the responses of single and dual receiver systems recorded in the same borehole. Figure
shows the mud signal is removed by two-receiver system.
110
TRACK
The distance between the receivers, or span, determines the tool’s vertical resolution – i.e., the thinnest
bed that can be detected by the measurement. As a rule, tool resolution equals the span between the
receivers. Smaller the spacing in between the receivers, the greater the details are (thin bed resolution).
The distance to the farthest receiver is limited by the power of the transmitter and the level of noise in
the borehole. However, the distance between the first receiver and the transmitter should be large
enough that the first arrivals are the refracted compressional waves that travelled a considerable
distance through the formation.
It is the minimum spacing required to ensure that first arrivals are compressional waves.
CYCLE SKIPPING
The acoustic waves attenuates (reduce, retard, weaker) as it propagates through the formation and
borehole environment. If the wave is attenuated beyond the threshold of the receiver, the receiver
misses the first arrival and detects a later event. In this case, log display an abnormally longer travel
time.
111
For 1 ft spacing, travel time increases in multiple of 33 and for 3 ft spacing, travel time
increases in multiple of 11 .
Cycle skipping commonly occurs in series of thin beds of different velocities, gas sands, mud,
poorly consolidated formations, and fractured formation.
Cycle skipping yield an incorrect reading.
It is useful for gas bearing and fractured formations indication.
Sonic wave is attenuated by refraction as it crosses the fissures.
The main short coming of the two receiver system is that in case of bore-hole irregularities or tool
tilting, the mud path travel time for both receivers does not remain the same due to which error is
generated as mud path error is not completely removed, which can be seen on the log as sharp spikes or
horns. It is overcome by using the dual transmitter system.
To remove this, two transmitters and four receivers are used. It is commonly run with 3-ft spacing
between each transmitter and its near receiver and with 2-ft receiver span (between two receivers). The
diagram shows the effect of tool tilt on the upper half of the array (a transmitter and two receivers) is in
the opposite direction of the effect on the lower half. An average of the measurement from the two
halves (upper and lower halves) cancels the error.
112
ADVANTAGES OF LONG SPACING SONIC SYSTEM OVER STANDARD TYPE
Long spacing sonic system provides a good-quality log in large diameter holes drilled in slower
formations.
Long spacing sonic tools are used to overcome the limitation caused by a large borehole and the
presence of an altered zone in seismic applications of sonic logs.
DEPTH OF INVESTIGATION
From Zaki
The acoustic tool’s depth of investigation vary with the wavelength ‘λ’, which is related to the formation
velocity ‘v’ and signal frequency ‘f’ by the equation:
In sedimentary rocks, ‘v’ varies from 5000 to 25000 ft/sec. For a wave frequency of 20kHz, λ varies from
0.25 to 1.25 ft. Laboratory experiments show that a thickness of at least thrice of the wavelength is
necessary to support a compressional wave over an appreciable distance. Hence for a 20kHz wave,
depth of investigation varies from 0.75 ft for soft formations to 3.75 for hard formations.
From Rider
The path of the compressional waves detected by sonic tools is essentially along the borehole
wall with very little penetration, generally between about 2.5 cm to 25 cm.
Increasing transmitter-receiver distance on a sonic tool increases the compressional wave
penetration, which was the reason for the introduction of the long spaced sonic sonde. The
increase in investigation occurs because the compressional wave in the damaged zone is slower
than the wave in the undamaged formation.
113
In the reverse physical situation, in gas zones where invaded formation, with fluid saturation,
has a faster velocity (slower) than the virgin formation saturated with gas.
Cycle skipping
Noise spikes
In extremely poor holes, cycle skipping occurs. This is the effect when the first compressional
wave arrival is too attenuated (weak) to activate the receiver, which is only tripped by a
subsequent arrival; the recorded time is therefore too large (interval transit time too large).
The reverse situation occurs when noise signals trip a receiver. This causes noise spikes on the
log and is found in hard formations such as limestone.
The path of the compressional waves detected by sonic tools is essentially along the borehole
wall with very little penetration, generally between about 2.5 cm to 25 cm.
114
QUANTITATIVE USES
Porosity calculation
When a formation has on average, a uniform distribution of small pores and is subjected to a heavy
confining pressure the there is a simple relationship between velocity and porosity
( )
This equation simply indicates that the transit time measured by the tool is the sum of the time spent in
solid matrix and the time in fluid, it is called time average relationship.
115
QUALITATIVE USES
Lithology identification
The velocity of the common sedimentary rock types is rarely diagnostic of lithology
There is too much variation within rock and too much overlap between types. Generally high velocity
most likely in carbonates, middle velocity in sand and low velocity in shale.
Texture identification
Way in which sound travel through the formation is associated with matrix, grain size distribution,
shapes and cementation in between grains. These all are associated with texture. For example, interval
of debris flow which has chaotic texture and no bedding i.e. we can’t differentiate beds and shows high
travel time than normal bedding.
Fracture identification
The sonic log porosity is probably only that due to matrix, and does not include fracture porosity. This is
because the sonic pulse will follow the fastest path to the receiver and this will avoid fractures.
Comparing sonic porosity to global porosity (porosity obtained from neutron or density) should indicate
zones of fracture.
Compaction
As sediment becomes compact, so its velocity increases. Over thick shale intervals, there is a regular
increase in velocity downwards due to compaction.
116
Uplift and erosion
The sonic is used to estimate uplift. Well ‘A’ shows normal compaction (curve A). Well ‘B’ shows over
compaction relative to well A at the same depth because of uplift and subsequent erosion (curve B). The
amount of uplift and erosion (Ea) is the vertical (depth) distance between curve A and curve B.
117
Correlation
It is difficult to do correlation in short interval but some of the variation in large section can be
correlated with other well log that shows the same variation.
A decrease from the normal compaction trend indicates overpressure which is normally due to increase
in porosity of shale.
118
DENSITY LOG
INTRODUCTION (USES):
The density log is the measure of bulk density of the subsurface formations along the depth. The
bulk density includes the density of the formation matrix and the density of the pore fluid. The
measurement of density helps to evaluate the porosity of the formation. Moreover the density tool also
measures the index of photoelectric absorption ‘Pe’ which helps to evaluate the lithology.
A radioactive source applied to the hole wall emits medium-energy gamma rays into the formation so
these gamma rays may be thought of as high velocity particles which collide with the electrons in the
formation. At each collision the gamma ray loses some of its energy to the electron, and then continues
with diminished energy. This type of interaction is known as Compton scattering. The scattered gamma
rays reaching the detector, at the fixed station from the source, are counted as an indication
of formation density. Lower the gamma ray counts detected by the detector, more the
interaction has occurred and hence more is the density.
DETAILED WORKING OF THE TOOL:
The tool consists of a transmitter that contains Cesium 137 source to generate medium energy
0.662 MeV gamma rays that enter into the formation. We know that gamma ray in the formation can
undergo three types of interactions such that pair production, Compton scattering and photoelectric
absorption. Since pair production requires energy of gamma ray greater than 10MeV, therefore in the
case of density log, pair production will not occur.
So, the two processes that will occur are the Compton scattering (helps to find bulk density and
porosity) and photoelectric absorption (helps to find ‘Pe’ and thus lithology). Initially when gamma ray
enter into the formation, it performs the Compton scattering such that they will strike with the
outermost electron of an atom, transfer some energy to it and then scatters with some low energy.
These low energy gamma rays can either leave the formation and are detected by the first detector or
gets absorbed in the inner orbital electron (photoelectric absorption) of another atom if their energy is
less than 0.2MeV, transfer their complete energy to it and expel it out with some kinetic energy
detected by the second detector.
In above explanation, we have discussed the two detectors, basically in the density tool there
are two sodium iodide detectors:
Short space detector; it measures the energy related to the Compton scattering in terms of count rate.
It is coated with cadmium shield which absorbs all gamma rays <140keV and thus help to find the bulk
density.
119
Long space detector; it measures the energy related
to both Compton scattering and photoelectric
absorption and thus help to find both bulk density and
index of photoelectric absorption (Pe) unit is
barns/electron.
120
POROSITY DETERMINATION BY BULK DENSITY:
The relation used to find porosity by bulk density from density log is:
121
Since, the depth of investigation of density tool is very small therefore it measures the bulk density in
invaded zone. If the formation is water bearing then fluid density will be simply mud density or filtrate
density as invaded zone will contain only filtrate of the mud (the mud could either be fresh water or salt
base mud).
If the formation contains hydrocarbons, then invaded zone will contain residual oil saturation and mud
filtrate saturation then total fluid density will be calculated as:
( )
But since the fluid density calculated from this gives the porosity result that is almost equal to the
porosity calculated by taking fluid density equal to mud filtrate density therefore we do not find fluid
density in this manner and simply use the mud filtrate density.
For gas bearing formation, we cannot find porosity by this method.
Example 8.1-8.5 in workbook
122
NEUTRON LOG
INTRODUCTION:
Since neutrons exhibit great penetration power so they are important in well logging. And
moreover, since these can be absorbed with very high efficiency by hydrogen atoms which are the most
common part of fluid (water of hydrocarbon) present in the pore spaces so they help to evaluate the
porosity of the formation.
PRINCIPLE:
123
Through Spontaneous reaction:
In this when alpha emitting particles such that plutonium, radium, etc emit alpha particles which
when strike with small atom nuclei such that beryllium, boron and lithium and emit neutron particles.
The neutrons produced by this type of reaction has maximum energy of 13 MeV, the neutrons produces
in this case have energy between the interval curve shown in figure on right side with peak of 4MeV. A
neutron produced from this type of source is called fixed source because the reaction is continuous and
uncontrolled.
In this method we bombard the low atomic number atoms by positive ions moving under the
influence of applied electric field. Due to applied electric field, we can control the speed of charges
therefore in this type, high mono-energy neutrons are produced with energy of 14MeV. We mostly use
this type for emission of neutrons in sources.
The neutrons moving freely are classified on the basis of their kinetic energy:
Fast (>100keV)
Intermediate (100 to 100keV)
Slow(<100eV)
Slow neutrons are further divided into epithermal (0.1-100eV) and thermal (0.025eV). Thermal
neutrons are those that are in thermal equilibrium with the surroundings, the value of their
energy such that 0.025eV is the root mean square translational KE of gas molecules at 20oC.
NEUTRON INTERACTIONS
124
Now neutrons once produced enter into the formation and strike with nuclei of the atoms present
in the formation and lose its energy during the interaction. The interactions that are possible are of
following types:
Moderating interactions
o Elastic collision
o Inelastic collision
Absorptive reaction
Moderating interaction:
This type of interaction mostly occur when the neutron has energy above the thermal neutron
energy value, in this type, the neutrons collide with the either elastically or inelastically with the
nuclei of various atoms in the formation and lose its energy to lower values, until the neutron
energy becomes equal to the thermal energy.
Inelastic collision:
In this type, the neutrons when collide with the nucleus of any atom loose some of their
energy to them and scatter with some low energy. The collision is not head to head collision and
more over as energy is transferred to the nucleus, the nucleus excites and emits gamma ray. The
neutron that is striking the nucleus must have energy above the maximum threshold energy of the
nucleus that is required to excite it. For example, a gamma ray of 6.09MeV appears when neutrons
scatter inelastically from an oxygen nucleus. This usually happens when neutron strikes a nucleus
which has mass far greater than the mass of neutron such that oxygen and others because the
greater the mass difference between the neutron and the neutron with which it is colliding, the
lesser will be the energy lost.
Elastic collision:
If the collision is head to head and the colliding neutron has energy above the thermal
energy then neutron will bounce back with some low kinetic energy, while the KE lost is transferred
to the nucleus of the atom to which the neutron had collided, the nucleus moves and strike with
some other nucleus and loose its energy until it is absorbed and gamma ray is emitted. This happens
when the neutron strikes with the nuclei of masses equal to the mass of neutron for example
hydrogen atom. In this case most energy is lost by neutron.
The neutron that is scattered or bounced back with low energy will again go into either of elastic
or inelastic collision until the energy reaches to its domain of thermal energy and finally the neutron
is absorbed.
In short, elastic collision occurs when the neutron strikes with a nucleus of the mass identical to its
own mass such that hydrogen nucleus and in this most energy is lost. Inelastic collision occurs when
125
the neutron strikes heavy nuclei such that oxygen, silicon, etc and in this case energy lost is not of
much extent, both the collisions tend to lower the kinetic energy of the neutron and thus slow it
down with in a length called as slowdown length and at the end they convert into thermal neutrons.
Absorptive reaction:
Absorption of neutron in a nucleus can occur at any energy but capture cross-section σc
varies inversely with the neutron energy so most absorption occur at very low energy that is thermal
energy neutrons have the most tendency to be absorbed by a nucleus, so we take reference of
thermal energy neutrons. Now it is not always possible that all the neutrons of thermal energy when
bombarded to any atom’s nucleus are all absorbed that is because this depends on the tendency of
the element. For example if 100 thermal neutrons are bombarded to hydrogen atom then all 100
can be absorbed how ever in case of carbon only 30 can be absorbed. Chlorine has also very high
tendency to absorb the thermal neutrons after hydrogen.
Neutron diffusion:
The neutrons introduced in an environment if have energy above the thermal level then
perform the moderated interaction, the energy will reduce, if resulting energy of neutron is above
thermal then it will again perform the moderating interaction but if the energy is equal to thermal
energy then it will be absorbed.
The neutron population at any point in the medium depends upon the capture and diffusion
processes. Capture reduces neutron population, but diffusion can increase or decrease the neutron
population.
TYPES OF DETECTORS
We are studying the detectors and not the tool types because the tool type depends on the
detector type as the source will be common in all tools which is meant to generate neutrons. The
tool emits high energy 4.5 MeV neutrons from a radioactive source. They move very fast and their
energy is related to their speed. They are called as fast neutrons. The neutron sources are mixture of
two elements 1) A source of alpha radiation such that polonium or radium and 2) the targeted
element such that Beryllium.
The alpha particle reacts with beryllium and produces a carbon atom, a neutron and a gamma ray.
It is meant to detect the captured gamma rays by a Scintillation tube. A tool containing this type
of detector is known as neutron gamma (N-G) tool. It has relatively large depth of investigation as
compared to neutron detecting tool. It has the disadvantage that the detector can also read the
counts of gamma rays produced in the source during the formation of free neutrons and the gamma
rays that are originating from the formation due to presence of natural radioactive elements, certain
arrangements are needed to filter these noises out and get the pure counts of captured gamma
126
rays. The gamma rays produced have to travel through the borehole fluid and are affected by the
bore-hole environment. For example, the presence of chlorine in the drilling fluid or formation
water cause the count rate to increase because chlorine is an efficient thermal neutron absorber
and emits several gamma rays. The thermal neutrons may reach the detector unabsorbed and gets
absorbed in the material of detector and emit gamma ray so the counts will also be including a
thermal neutron component and is called a hybrid measurement, in practice it is difficult to have a
pure NG tool.
Slow neutron detectors are also available. They are predominantly sensitive to thermal neutrons
but also have varying sensitivities to captured gamma rays, thus giving a hybrid measurement. A tool
using this type of detection is called thermal neutron tool.
Since in most cases formation has the moderate quantity of thermal neutron absorbers, so the
spatial distribution of captured gamma rays and thermal neutrons are essentially the same so the
thermal neutron detector and NG tool are similar in construction and show approximately same
response in such kinds of formations but their responses will be very different if thermal neutron
absorbers such as chlorine, hydrogen, boron are greater in quantity in formation that will absorb
more thermal neutron and give large captured gamma rays so thermal neutron tool will give low
count rate but NG tool’s response will be great.
Some detectors respond to epithermal neutrons and ignore both thermal and gamma rays. They
are basically the thermal neutron detectors that are covered with cadmium sheath to absorb the
thermal neutrons and only allow the thermal neutrons to pass thorough and get detected.
Epithermal neutron detectors count rates are less sensitive to lithology and formation salinity as
compared to the thermal neutron tools. These tools respond hydrogen containing formation and
subsequently derived porosity value.
The NG tool measures the counts of captured gamma rays. The tool is available for different
spacings among which we have to select the best for given bore-hole conditions. On the log the
response is shown as units of API. One API unit is equal to 1/1000 times the difference between the
instrument zero (instrument response to the zero radiation) and the response of the instrument in
front of the 6ft thick Indiana limestone with hole size of 7 7/8inch, fresh water in the pores, 75 oF
and atmospheric pressure.
In order to calibrate the tool to porosity, the pit of Houston university can be used such that we
know the porosity value of Indiana limestone and more over the tool will give us the API value in
front of this zone then in this we will get a relation between API value and porosity but this is only
valid for ideal conditions as discussed earlier. For conditions being deviated from this, we should
127
apply corrections for bore-hole environment by using chart (each chart will be for given tool
spacing) and then measure the porosity (corrected porosity). The porosity we read from the
correction curves will be the limestone based porosity, for sandstone and dolomite porosities, we
have to use the cross plots.
Problem 9.1:
The N-G log of following figure was obtained with a GNT tool with a 19 ½ inch source to detector
spacing in an 8 inch open hole filled with 13 lbm/gal freshwater based mud. The average
temperature over the interval shown by the log is 200oF. Calculate the porosities in zones A and B.
Solution:
128
129
The porosity values based on limestone are:
In zone A:
Porosity is 16%.
In zone B:
Porosity is 3%.
In zone C:
The value of porosity lies above 50% which is of course the apparent resistivity. This high
porosity value comes from the fact that low log response is there and this is because the
environment is of shale where thermal neutrons are greatly absorbed in the formation due to high
porosity and the captured gamma rays reaching to the detector are very low that is why gamma ray
log is showing high value in this zone.
It detects the epithermal neutrons and rejects the thermal and captured gamma rays. The larger
the count rate of epithermal neutrons means very few of them are converted into thermal neutrons
and gets absorbed which shows low porosity value of the formation.
Its spacing is short. To enhance the depth of investigation, the source and the detector are
mounted on a directionally sensitive pad that is pressed firmly against the bore-hole wall with a back
up shoe. This design minimizes the bore-hole effects and provides a mean of obtaining a caliper log.
The log gives the direct value of the porosity as the panel directly converts the count rate
response to the porosity using calibration charts that are produced by observing the log responses
(as count rates) against the formation samples of known porosities. The panel by using these curves
gives the porosity that is true for the ideal environment having the following conditions:
If the actual conditions are not the same then corrections are needed that are calculated using
different charts, since corrections are very small due to exclusion of the borehole effects, therefore
the panel itself applies the correction and the log gives the corrected porosity to the actual
environment. The porosity we will get is the porosity based on limestone.
130
Problem 9.3:
A 20% porosity was determined from the SNP calibration curves. If the actual measurement
conditions are a borehole diameter of 10 inch, a 16 PPG mud, salt saturated brine mud as drilling
fluid, a formation temperature of 175oF and pressure of 5200 psi, salinity of formation fluid is
250000 ppm. Estimate the order of magnitude of the correction applied by the panel.
131
Solution:
132
Using figures given above, the borehole correction is -1%, salinity correction is +2.5%, mud-weight
correction is +3%, and temperature and pressure correction is 0.5%. This represents a total correction of
+5%.
133
The borehole size of 7 7/8 inch
Fresh water in the bore hole and formation water
No mud cake
Tool stand off
75oF temperature
Atmospheric pressure
Tool eccentered in the hole (remain attached with the bore hole on one side)
The actual conditions may deviate from the above conditions so we use series of correction charts to
calculate the corrections and then add them
algebraically to get total correction factor.
The charts are shown in the figures of
example 9.4.
134
Problem 9.4:
A 1980 CNL run in a combination with an FDC indicates a 28% apparent porosity in zone of
interest. Correct this value for borehole conditions. The measurement environments are as follows:
Mud cake thickness is 0.5 inch, mud weight 12 PPG, drilling fluid salinity of 100,000ppm, formation
water salinity is 150,000ppm, formation temperature of 225oF. Formation depth is 2100 ft and the tool
was run eccentric.
135
Chart B Correction for mud cake presence +2%
Chart C Borehole salinity correction +0.2%
Chart D Formation salinity correction -1.4%
Chart E Mud weight correction +1.7%
Chart F Standoff correction 0%
Chart G Pressure correction -3.4%
Chart H Temperature correction +4.3%
Total correction=+3.4%
If we want to determine the porosity of sandstone or dolomite from this value then we use cross plots
as shown below:
136
137
Neutron log should not be used to determine shale porosity.
138
139
140
141
DENSITY NEUTRON COMBINATION
If density-neutron overlaps then it indicates limestone filled with water
142
In case of positive separation, it indicates high water content shale
143
DENSITY AND NEUTRON LIMESTONE SCALES
If density log is on left as compared to neutron log then following conditions are possible
If density log is on right as compared to neutron log then following conditions are possible
1. Shale
2. Water filled Dolomite
144
145
DIPMETER LOGGING
It measure the dip of the formation
Measure fault, unconformities, etc.
Paleocurrent direction
Folds, fractures and in seismic line matching
146
IMAGE LOG
Image log is the measurement from the density image, resistivity image and acoustic image.
a) Acoustic travel time, which measure the time taken by the acoustic signal to travel to and
back from the bore-hole wall.
b) Acoustic amplitude, which measure the acoustic impedance of the formation.
The most common factors that affect the measurement in image log are:
Borehole rugosity
Mud type
Formation fluid
LWD images are less affected by bore hole invasion hence would offer better quality image because
during drilling the time in during which mud exposed to the formation is less and less invasion occurs
moreover caving in are also less but during wire-line logging the mud remain static for long time, more
invasion occurs.
Core marks
Fracture
Bedding plane
Stresses
Faults
Perforation marks
Dipping direction
147
PRODUCTION LOGGING
Purpose of production logging is to evaluate fluid flow inside and outside pipe and to measure
well flow profile.
Starts from early stage of drilling and last throughout life of well.
In non-technical words, production logging is “How much of what fluid is coming from where”.
During lost circulation, fluid invade into formation and cool it so when fluid shoots out from bit and
moves upward in annulus then in front of lost circulation, it becomes abnormally cool which can be seen
from temperature log.
148
Identification of well kick source and underground blowout
Kick is controlled flow of fluid into the well bore. In this condition, we shut BOP and pressure of well
bore increases rapidly causes formation above kick source to break down cause underground blowout.
In temperature log, temperature increases rapidly and differential temperature shows peak. Well kick
source is 17000ft because we have drilled up to that depth and underground blowout occurs at 15600ft
which is clear from the peak of differential graph.
After cementation job, curing cement is an exothermic reaction (heat liberates out) so by running
temperature log we can identify top of cement by increase of temperature.
149
Cement quality
To check cement quality, we run cement bond log. In cement bond log, acoustic waves travel from
emitter to detector. In case of bad cement job, acoustic waves attenuate less and velocity of acoustic
waves will be high and will be less and vice versa for good cement job.
150
EVALUATION OF GAS BEARING FORMATION
Porosity logs (density, neutron and sonic) have relatively shallow radii of investigation
Generally, gas solution near the well bore in all types of formation causes an increase in density
log and decrease in neutron .
The presence of gas results in an increase in sonic log porosity only in poor compacted sands,
which is the cause of most shallow sand and some abnormal pressure formation.
Using density and sonic log porosity values uncorrected for gas effect usually results in a low
apparent value of the formation factor ‘F’ due to high porosity. Thus, results in low apparent Ro.
Whereas use of uncorrected neutron porosity give an opposite effect from density and sonic
Gas – bulk density too low gives density porosity too high
Gas – lower count rate of neutron gives neutron porosity too low
The sonic log usually is not recommended because in addition to gas and shale effect and
compaction and secondary porosity effect can be present.
Three factors determine the response of porosity logs in gas bearing formations; actual porosity,
gas saturation and degree of shaliness.
151
more difficult. When shale and gas effects completely offset each other, a shaly gas-bearing formation
may look like a clean liquid filled formation as clear from zone C.
( )
( ) is density porosity reading in clean gas bearing
formation and is formation’s true porosity. It allows
the detection of gas zone provided that (formation
true porosity) of the given zone is known or can be
estimated. In a formation of uniform lithology and
porosity, the neutron log alone can indicate the
gas/liquid (gas/oil or gas/water) contact as in figure.
The identification is possible because of the bulk
density contrast between gas and liquid filled zones.
Laboratory tests show that presence of compressible fluid (gas) in the pore space of unconsolidated
sands greatly increases acoustic travel time and reduces velocity of acoustic wave. But the sand has
some degree of consolidation; the increase in travel time caused by the gas is lessened or completely
erased.
The dependence of the gas anomaly on degree of formation compaction makes the sonic log unsuitable
for gas detection in most cases. As with neutron and density logs, presence of shale in the formation
further complicates the problem.
152
SUMMARY OF ALL LOGS
1. RESISTIVITY LOG
A. Resistivity of formation depends upon
a) Formation water resistivity
b) Amount of water present
c) Pore structure geometry
B. Resistivity log evaluation
i. Rw
ii. F
iii.
iv. Rt
C. When Rxo > Rt (induction log because of oil based mud)
D. Focused current device is relatively for thin bed which gives high vertical resolution
E. Induction log is for
i. Formation resistivity measurement
ii. Work in non-conductive mud
iii. Use in pneumatic drilling
iv. Highly accurate in low resistive formation
2. SP LOG
Use to distinguish between permeable and impermeable formation
Bed thickness
Determination of Rw
Salinity factor
The flow of ions due to diffusion potential and membrane potential
It measures the natural electric potential between borehole and formation
3. POROSITY MEASUREMENT BY
Sonic log
Neutron log
Density log
To determine
Improve correlation and interpretation of seismic record
Identify high pressure zone
Estimate secondary porosity
Density measurement
Transit time is inversely proportional to velocity and depends upon lithology and porosity.
153
4. NEUTRON LOG
Neutron log is to measure
I. Porosity
II. Detection of gas or light hydrocarbon
III. Lithology identification
Neutron interaction are:
a) Moderating interaction: are elastic or inelastic results to slow down neutron
(loss of energy)
b) Absorption interaction: results in disappearance of neutron
Neutron diffusion: They slow down and go through the intermediate, epithermal and thermal
energy level.
Maximum energy loss in hydrogen due to less number of collisions.
Chlorine causes the count rate increase because of efficient thermal neutron. Neutron absorbed
that emits gamma ray.
NG tool detect the gamma ray by ignoring thermal and epithermal neutron.
c) Proportional counter
Slight modification of GM counter
100 times efficient then GM counter
154
d) Scintillation counter
Fluorescent crystals and photomultiplier used
Gamma ray exposed to crystal
Strike with cathode result in photoelectric emission
Striking again with anode at different potentials
At each anode, electron multiply (secondary electron)
Large number of electrons at last anode
Efficiency is 50 to 60%, best use, fast reaction and temperature sensitive
6. IMAGE LOGGING
Types of instruments are; resistivity, ultrasonic and density
Uses are:
a) Core and perforation knowledge
b) Fracture and fault identification
c) Bedding plane
d) Very bright area shows gas sand
e) Bed thickness
f) Stress direction
g) Packer marks
Temperature log
It measures geothermal gradient
Thermal conductivity (rock with high TC has low geothermal gradient and vice versa)
The difference in gradient is related to the difference in heat flow value in different zones
Common method to temperature correction is Horner plot
Use in correlation, Hydrocarbon maturation, fluid movement, overpressure zone
Most commonly used in production logging
Caliper log
Caliper measure hole size VS. depth
Used in track 1
Estimate borehole volume
( )
( ) ( )
155
LAB SESSION 1
TEMPERATURE LOG
156
LABORATORY SESSION 1(a)
OBJECT:
SOLUTION
( )
(c)
( )
( )
157
Hence, the deviation in calculated gradient is only and the deviation in calculated formation
temperature is . The sensitivity to assumed surface temperature in the range of 60 to 800F is
very small in this case. It generally decreases as total depth increases.
158
LABORATORY SESSION 1(B)
OBJECT:
Drilling of a 16200 ft borehole was finished at 1:30 a.m. The mud was then circulated until 4:00 a.m.
Three logging tools were run to TD successively. The tool type, the time the tool moved off bottom,
and the maximum recorded temperatures are shown below: (EXAMPLE 4.5 ZAKI)
SOLUTION
The circulation of mud is stopped at 4:00 and from that time, temperature of mud Tm starts rising until it
reaches formation temperature Tf but it takes too much time. So, we calculate mud temperature Tm at
time 𝞭t, where 𝞭t is time elapsed since circulation stopped. First induction tool is run in which maximum
indicating thermometer is present which measures the maximum temperature of mud beside the
formation and we take it off from bottom formation at 12:15 so, maximum temperature is encountered
at that time because mud temperature in the upper formation is less. It means mud temperature rises
to 2410F in 𝞭t=12:15-4:00=8:15=8.25hours. Similarly we run density log and measure maximum
temperature of mud at 15:00 and then take it off from bottom. Time elapsed for that temperature of
mud is time since circulation stopped. Similar procedure is done for neutron tool.
159
Tm becomes equal to Tf when and which is possible only when
(c) Let Sw1 and Sw2 be the water saturation values calculated with 2410F and 3020F respectively.
( )
( )
160
( ) ( )
( )
( ) ( )
( )
161
LAB SESSION 2
RESISTIVITY LOG
162
LABORATORY SESSION 2(A)
OBJECT:
SOLUTION
a)
b) Using resistivity nomographs for NaCl solutions, line joining the two points that represent 77 0F
and 0.05𝝮m intersect the salinity scale at about 170 kppm (170,000 ppm).
163
Now, to obtain concentration of NaCl in g/L, use nomograph for concentration and density of NaCl
solutions
c) In Fig 1.7, line that joins the two points representing 2000F and 170,000ppm intersects the
resistivity scale at a value of 0.02𝝮m.
( )
( )
164
LABORATORY SESSION 2(B)
OBJECT:
Determine the formation resistivity factor of two US Gulf Coast sandstone formations with porosities
of 15% and 25% respectively? (EXAMPLE 1.7 ZAKI)
SOLUTION
Because there is no mention of core measurements a, m so generalized equation is used. Following are
the generalized equations available:
( ) ( )
( ) ( )
( ) ( )
( ) ( )
CONCLUSION
For formation displaying relatively higher porosity, four values of F are close to each other so
any value can be used.
For formation displaying relatively lower porosity, four values of F show high deviation. The
value of 36.6, calculated from Humble equation, is the least believable if we take into
consideration the number of samples used to derive different equations. Now, the question of
which value to choose is avoided by reporting range of values i.e.,
F = 22.8 to 30.1 @ Φ=25%
It is because F is usually used to calculate other parameters.
165
LABORATORY SESSION 2 (C)
OBJECT:
The resistivity of a clean, consolidated sandstone rock is 10.0 𝝮.m. if the rock porosity and formation
water resistivity are 15% and 0.05 𝝮.m respectively, determine whether the rock is hydrocarbon-
bearing and, if so, to what extent. (EXAMPLE 1.8 ZAKI)
SOLUTION
( ) ( )
166
LABORATORY SESSION 2 (D)
OBJECT:
Find the resistivity value at 2180Fwhen a surface measurement of 0.9𝝮.m at 800F? (EXAMPLE 4.2 ZAKI)
SOLUTION
( ) ( )
167
LABORATORY SESSION 2 (E)
OBJECT:
A given fluid loss, differential pressure, mud cake, and exposure time indicate that 20 lit of mud
filtrate invades into a porous, permeable oil bearing formation, if formation thickness is, h= 10 ft,
residual oil saturation Sor= 25% and bore hole diameterdn=8in. Calculate diameter of invasion, di=? if
porosity is (a) 40% (b) 10% (EXAMPLE 4.3 ZAKI)
SOLUTION
h=10ft=120in dn
V=20lit=20dm3 h
1dm3=1000cm3 di
( )
( ) ( )
( ) ( )
( ) ( )
CONCLUSION
Taking porosity as 10% and invaded volume same as above is not practically possible. In actual, less
porosity means less permeability and less invaded volume. If invaded volume is same then in case of
high porosity, invaded volume will not go far and less diameter of invasion is present.
168
LABORATORY SESSION 2 (F)
OBJECT:
A cylindrical core sample of a well consolidated sand is completely saturated with a synthetic brine of
50,000 ppm salinity. At 1040F, the resistance of the core (from end to end) is 980 ohms. The core is 3.5
inch in diameter and 12 inch long. Compute its porosity by both Archie and Humble equations?
(EXAMPLE 11.2 (CARL GATLIN))
SOLUTION
( )
( )
Archie equation
For m=2
Humble equation
( )
169
LABORATORY SESSION 2 (G)
OBJECT:
Suppose that the core sample of example 11.2 is now partially saturated with oil so that its resistivity
is 20𝝮.m. What is its oil saturation? (EXAMPLE 11.3(CARL GATLIN))
SOLUTION
( )
( )
( ) ( )
170
LABORATORY SESSION 2 (H)
OBJECT:
Given:
Required Sw
SOLUTION
171
Archie equation
( )
( ) ( )
172
LABORATORY SESSION 2 (I)
OBJECT:
SOLUTION
( )
( )
173
LABORATORY SESSION 2 (J)
OBJECT:
A practical example is given by taking brine analyzed chemically to have 50,000 ppm total solids that
include 10,000 ppm Na, 16,000 ppm Cl, 7,000 ppm Mg, 5,000 ppm Ca, and 12,000 ppm SO 4. Determine
NaCl equivalent concentration and Rw @ 2500F?
SOLUTION
The equivalent NaCl concentration chart is used for total solids concentrations > 1,000 ppm.
The concentration of each ion is then multiplied by the weighting multiplier and the products for all the
ions are added together to determine the equivalent NaCl concentration.
∑ ( )
The multipliers for Na and Cl are 1, multiplier for Mg is 0.9, multiplier for Cais 0.78 and for SO4is 0.36
( ) ( ) ( ) ( ) ( )
174
Figure 2
175
LABORATORY SESSION 2 (K)
OBJECT:
Calculate the resistivity of NaCl at 2500F when salinity is 2500ppm using pi-chart?
SOLUTION
This answer is also verified from figure (2) used in previous lesson shown by green line.
176
LABORATORY SESSION 2 (L)
OBJECT:
Given the values below, calculate water saturation, assuming zero shale volume, and values of n=2,
m=2, and a = 1, Rw= 0.05 @ formation temperature.
SOLUTION
Using formula
( ) ( )
Rt Φ Sw
55 22 14
20 12 42
220 8 19
8 18 44
5 10 100
177
LABORATORY SESSION 2 (M)
OBJECT:
Given the following calculated results for Phi and Sw, determine the depth intervals of water-free
production, the transition zone, and the oil-water contact. Indicate each with brackets around those
depth intervals
SOLUTION
Water free oil: It is because water saturation is nearly constant and minimum which indicates connate
water saturation.
178
LABORATORY SESSION 2 (N)
OBJECT:
SOLUTION
For Rw
( )
( )
For Sw
( )
179
Water resistivity Rw=0.02 𝝮.m
Formation resistivity Rt =400 𝝮.m
a=1, m=2, n=2
( ) ( )
180
LABORATORY SESSION 2 (O)
OBJECT:
SOLUTION
The example on the chart shows entries of 10 and 1.4, which cross one another at a point where di = 39
in., Rxo/Rt= 18.5, and Rt/RILD= 0.95
This calculation demonstrates that RILD is not seriously affected by an apparent 39-in. (1.0-m) invasion
diameter.
181
LABORATORY SESSION 2 (P)
OBJECT:
SOLUTION
Therefore,
182
LABORATORY SESSION 2 (Q)
OBJECT:
Determine resistivity of brine at 2500F when resistivity of brine at 2000F is 0.1 𝝮.m?
FORMULAE
( )
( )
SOLUTION
( )
( )
183
LABORATORY SESSION 2 (R)
SOLUTION (a)
Multipliers are;
( ) ( ) ( ) ( ) ( )
( )
184
Resistivity at different temperatures
SOLUTION (b)
At 1000F
185
At 1500F
At 2000F
186
LABORATORY SESSION 2 (S)
A drilling mud has resistivity of 0.3 at 2000F, which is BHT at 10,000ft. In interpretation of log at
7000ft, what Rm should be used? The Tf of the area is 800F.
SOLUTION
( ) ( )
187
LABORATORY SESSION 2 (T)
SOLUTION
( )( ) ( )
188
LABORATORY SESSION 2 (U)
Given that:
(a) Compute ‘F’ for this zone and its porosity by Archie and Humble
(b) Compute ‘Sw’ by Archie for n=2
SOLUTION (a)
( )( ) ( )
Determination of porosity
1. Humble
2. Archie
SOLUTION (b)
( ) ( )
189
LABORATORY SESSION 2 (V)
From core analysis, the porosity of well consolidated sand is known to be 20%, formation water
resistivity Rw from actual sample is 0.04 at Tf. From electric log, Rt = 80𝝮m. Estimate Sw for this
zone for m=2.
SOLUTION
( )
( ) ( ) ( )
190
LAB SESSION 3
SP LOG
191
LABORATORY SESSION 3 (A)
OBJECT:
Thickness =32 ft
SP = 20 x 5 =100 mV
Rm =? ,Tf = ? , Rw =?
Ts = 700F
SOLUTION
( )
192
At SP = 100mV and Tf = 1720F
K=82
( )
For SSP
( )
( )
( ) (For confirmation)
For Rmf
For Rwe
( ) ( )
193
( )
For Rw
At ( )
For salinity
Salinity =70,000ppm @
194
195
LABORATORY SESSION 3 (B)
SOLUTION
K=80
( )
For SSP
( )
( )
196
( ) (For confirmation)
For Rmf
For Rwe
( ) ( )
( )
For Rw
At ( )
197
LAB SESSION 4
GAMMA RAY LOG
198
LABORATORY SESSION 4 (A)
OBJECT:
Sands and shales represent the interval shown in following figure. Determine Vsh (Volume of shale)
for both Tertiary and Mesozoic ages in zones A to I indicated by log?
SOLUTION
( )
( )
( )
( )
199
LABORATORY SESSION 4 (B)
OBJECT:
A sequence of carbonates and shales is given. Determine Vsh for both Tertiary and pre-Tertiary
(Mesozoic) in zones A through F?
SOLUTION
( )
( )
( )
( )
200
LAB SESSION 5
NEUTRON LOG
201
LABORATORY SESSION 5(A)
OBJECT:
The N-G log of following figure was obtained with a GNT tool with a 19 ½ inch source to detector
spacing in an 8 inch open hole filled with 13 lbm/gal freshwater based mud. The average
temperature over the interval shown by the log is 200oF. Calculate the porosities in zones A and B.
SOLUTION
202
203
The porosity values based on limestone are:
In zone A:
Porosity is 16%.
In zone B:
Porosity is 3%.
In zone C:
The value of porosity lies above 50% which is of course the apparent resistivity. This high
porosity value comes from the fact that low log response is there and this is because the
environment is of shale where thermal neutrons are greatly absorbed in the formation due to high
porosity and the captured gamma rays reaching to the detector are very low that is why gamma ray
log is showing high value in this zone.
204
LABORATORY SESSION 5(B)
OBJECT:
A 20% porosity was determined from the SNP calibration curves. If the actual measurement
conditions are a borehole diameter of 10 inch, a 16 PPG mud, salt saturated brine mud as drilling
fluid, a formation temperature of 175oF and pressure of 5200 psi, salinity of formation fluid is
250000 ppm. Estimate the order of magnitude of the correction applied by the panel.
SOLUTION
205
Using figures given above, the borehole correction is -1%, salinity correction is +2.5%, mud-weight
correction is +3%, and temperature and pressure correction is 0.5%. This represents a total correction of
+5%.
206
LABORATORY SESSION 5(C)
OBJECT:
A 1980 CNL run in a combination with an FDC indicates a 28% apparent porosity in zone of
interest. Correct this value for borehole conditions. The measurement environments are as follows:
Mud cake thickness is 0.5 inch, mud weight 12 PPG, drilling fluid salinity of 100,000ppm, formation
water salinity is 150,000ppm, formation temperature of 225oF. Formation depth is 2100 ft and the tool
was run eccentric.
207
SOLUTION
Total correction=+3.4%
If we want to determine the porosity of sandstone or dolomite from this value then we use cross plots
as shown below:
208
LAB SESSION 6
SONIC OR ACOUSTIC LOG
209
LABORATORY SESSION 6(A)
OBJECT:
A compacted sandstone formation of interest, detected by an old single-receiver sonic log, displays a
slowness of 90 . Estimate the formation porosity if the bit is 7 3/8 inch, tool diameter is 3
inch, centralizers were used, and the tool spacing is 6 ft.
DATA
( )
SOLUTION
( )√ ( )
( )
( )√ ( )
210
LABORATORY SESSION 6(B)
OBJECT:
Estimate the average porosity of the thick sand interval shown on the sonic log of figure
SOLUTION
211
LAB SESSION 7
DENSITY LOG
212
LABORATORY SESSION 7(A)
OBJECT:
The density porosity is usually calculated assuming f g/cm3Compare this apparent Porosity, a , to
the true porosity, t , that correspond to a bulk density of 2.1 g/cm3 in the following environments: (1) a
water-bearing sandstone invaded by a mud filtrate of 1.05 g/cm3 density, (2) a 0.8 g/cm3 oil-bearing
sandstone characterized by Sor = 30%, and (3) a low pressure gas bearing sandstone characterized by
30% residual gas saturation.
SOLUTION
( )
t = ( )
= 0.34 or 34%
Then t ≈ a.
( )
And t = ( )
= 0.32 or 32%
Again t ≈ a
( )
t = ( )
= 0.28 or 28%
And t < a ,
Thus except for gas bearing formations, it is reasonable to assume that mf = 1 g/cm3.
213
LABORATORY SESSION 7(B)
OBJECT:
A water bearing sandstone displays a 2.1 g/cm3 bulk density. Calculate the density porosity using a
matrix density of 2.68 g/cm3. Compare this value to that calculated with the normally assumed matrix
density of 2.65 g/cm3.
SOLUTION
( )
a = = 0.33 or 33%
( )
( )
t = ( )
= 0.345 or 34.5%
This represents a deviation of -1.5 porosity units and a relative error of about -4%.
214
LABORATORY SESSION 7(C)
OBJECT:
In answering the following questions, refer to the log of fig 8.16 obtained in a borehole drilled with
freshwater based mud.
a) What are the values of ρb, Δρ , gamma ray, caliper and φD in zones A and B?
b) What is the true porosity value of zones A and B?
Solution (a):
S.N0 Quantitty Zone A Zone B
2 φD , % -6 12
3 Δρ, g/cm3 0 0
5 Caliper, in 8 8
215
Solution (b):
The porosity curve of zone A was calculated with freshwater filled sandstone parameter (see log
heading). The log displays a negative apparent porosity of -6% next to Zone A indicates that a negative
value results from the fact that the measured ρb is greater than the assumed ρma. .It follows that he true
matrix density is greater than the true sandstone density. It also greater than the density of limestone
(ρb = 2.75>2.71). Knowledge of lithology is required for accurate determination of the true porosity of
this zone.
Although the porosity of Zone B is positive, it can still be an apparent porosity value because the
lithology can be something other than sandstone. The logs indicate (1) that the borehole is regular and
drilled to gauge (bit size is most certainly 77/8in); (2) that gamma ray response is low; (3) that Δρ=0
indicating possible absence of mud cake, which in turn suggests a low permeability formation; and (4)
that litologies other than sandstone are present e.g.,Zone A.
These facts indicate that zone B is probably not sandstone. Its true porosity can be determined only
after accurate lithologic identification.
216
LAB SESSION 8
MISCELLANEOUS PROBLEMS
217
LABORATORY SESSION 8(A)
OBJECT:
Fig 11.7 and 11.8are IES and Formation Density Compensated (FDC) logs obtained in a development
well drilled to produce the oil sand located at 10,420ft. A formation water resistivity of 0.11Ω.m at 60·F
is listed in formation water catalogs. Mud and temperature data listed on the well heading are as
follows:-
Bottom hole temperature (BHT) =274·F at 11,300ft give a complete qualitative and quantitative analysis
of the oil sand of interest.
218
219
SOLUTION
The oil sand of interest is in the interval of 10,402 to 10,422ft. The SP curve indicates fairly clean sand.
The sand displays a uniform porosity indicated by the almost constant density log response. Hence, the
sand is evaluated whole. Average log response is used for the evaluation.
Net Pay Determination: - The sand is clean with no major impermeable streaks, so the net sand
thickness is equal to the gross thickness, 20ft (10,422-10,402). No water/oil contact is present because
the entire sand is resistive. The net ay is equal to the net sand thickness, 20ft.
Porosity Determination: - The density log displays an average reading of 2.25 g/ . The tool is
automatically compensated for mud cake effect, and the caliper indicates a regular borehole. The log
reading is reliable and can be used to estimate the porosity with Eq.8.11:
Ф=( - )/ ( ).
Consistent with porosity evaluation principles of Chap 8, values of 2.65 and 1.0 g/ are assumed for
and respectively.
Then,
Drilling Fluid Properties: - and at formation temperature can be calculated from Equations 1.11
or with Fig. 1.7.
220
From SP log:
Maximum SP
deflection =
= -50 mV.
221
From Water Catalogs: -
The value of 0.028 is retained for the evaluation because it was obtained by direct measurement and is
believed to be more representative.
Determination: - The deep induction tool displays an average apparent resistivity, of 19Ω.m;
Consulting Fig. 5.56 confirms the negligible borehole effect on a deep induction tool in a small borehole
effect free apparent resistivity is
= =19Ω.m
On the other hand, fig. 5.62 indicates a significant bed thickness effect resulting in bed thickness effect
free apparent resistivity = 32Ω.m. Use of Fig. 5.62 was dictated by the values of the resistivity of the
shale that surrounds the sand.
= =35Ω.m.
F=
=1.13( ) =13.3
And =F
=13.3(0.028) =0.37 .m
=( )
( )
And =1-0.1028=89.72%
222
LABORATORY SESSION 8(B)
Evaluate the top zone (Zone A) of the sand shown by the IES and sonic logs in figure in terms of the
presence of movable hydrocarbons. The sand is a Lower Tuscaloosa (state of Mississippi) formation
that produced oil from wells drilled in other favorable structural positions.
223
Rmf = 1.40 𝝮m @800F
Rmc = 0.75 𝝮m @2160F
Bit size = 6 ¾ inch
SOLUTION
However, the SP reading of -165mV and a reading of 98 indicate an abnormally low water
resistivity and high porosity. The possibility of oil presence, then, cannot be dismissed. Detailed quantity
interpretation has to be executed.
For Rw
The sand is less than 100ft from the bottom of the hole. It is reasonable to consider Tf = maximum
recorded temperature = 2160F. The sand is thick and apparently clean, so ESSP = ESP = -165mV
( )( ) ( )
224
( )
Porosity determination
Shale travel time dispalyed by the sonic log is less than 100 , indicating a normally compacted
formation.
and
( )
( ) ( ) ( )
A 68% water saturation is relatively high; however critical water saturation values as high as 75% might
exist under optimum rock properties. An estimation of Smo might help the evaluation.
( )
Smo technique requires an Rxo value to determine Sxo. A reliable Rxo value cannot be determined from
available measurements. The short normal, however, can provide a lower limit of Rxo because it is
expected to be affected, to some unknown degree, by the univaded zone resistivity; however the Rt is
small so the effect is also small.
( ) ( )
( )
( )
225
Although the interpretation failed to give a specific value for Smo, it definitely indicated movable oil
could be present. Only an actual conclusive test could give a conclusive answer. This well was perforated
from 10,494 to 10512 ft, and initial production was 110 BOPD with no water.
226